You are on page 1of 90

5000

Solutions Manual:

Chapter 5

7th Edition

Feedback Control of Dynamic


Systems
Gene F. Franklin
J. David Powell
Abbas Emami-Naeini

Assisted by:
H.K. Aghajan
H. Al-Rahmani
P. Coulot
P. Dankoski
S. Everett
R. Fuller
T. Iwata
V. Jones
F. Safai
L. Kobayashi
H-T. Lee
E. Thuriyasena
M. Matsuoka
J.K. Lee

2015 Pearson Education, Inc., Upper Saddle River, NJ. All rights reserved. This publication is protected by Copyright and written permission should be obtained
from the publisher prior to any prohibited reproduction, storage in a retrieval system, or transmission in any form or by any means, electronic, mechanical, photocopying,
recording, or likewise. For information regarding permission(s), write to: Rights and Permissions Department, Pearson Education, Inc., Upper Saddle River, NJ 07458.

Chapter 5

The Root-Locus Design


Method
Problems and solutions for Section 5.1
1. Set up the listed characteristic equations in the form suited to Evanss
root-locus method. Give L(s); a(s); and b(s) and the parameter K in
terms of the original parameters in each case. Be sure to select K so that
a(s) and b(s) are monic in each case and the degree of b(s) is not greater
than that of a(s).
(a) s + (1= ) = 0 versus parameter
(b) s2 + cs + c + 1 = 0 versus parameter c
(c) (s + c)3 + A(T s + 1) = 0
i. versus parameter A,
ii. versus parameter T ,
iii. versus the parameter c, if possible. Say why you can or can not.
Can a plot of the roots be drawn versus c for given constant
values of A and T by any means at all?
kI
kD s
c(s)
+
G(s) = 0: Assume that G(s) = A
, where
s
s+1
d(s)
c(s) and d(s) are monic polynomials with the degree of d(s) greater
than that of c(s).
(Note: The rst printing of the 7th edition
kI
had an error in the equation above where the
term above was
s
incorrectly stated to be kI (s):)

(d) 1 + kp +

i. versus kp
ii. versus kI
iii. versus kD
5001

2015 Pearson Education, Inc., Upper Saddle River, NJ. All rights reserved. This publication is protected by Copyright and written permission should be obtained
from the publisher prior to any prohibited reproduction, storage in a retrieval system, or transmission in any form or by any means, electronic, mechanical, photocopying,
recording, or likewise. For information regarding permission(s), write to: Rights and Permissions Department, Pearson Education, Inc., Upper Saddle River, NJ 07458.

5002

CHAPTER 5. THE ROOT-LOCUS DESIGN METHOD


iv. versus

Solution:
(a) K = 1= ;
a = s;
b=1
(b) K = c;
a = s2 + 1;
b=s+1
(c)

(d)

i. K = AT ;
a = (s + c)3 ;
b = s + 1=T
ii. K = AT ;
a = (s + c)3 + A;
b=s
iii. The parameter c enters the equation in a nonlinear way and a
standard root locus does not apply. However, using a polynomial
solver, the roots can be plotted versus c:
i. K = kp A;
a = s(s + 1= )d(s) + AkI (s + 1= )c(s) +

kD

b = s(s + 1= )c(s)
ii. K = AkI ;
a = s(s + 1= )d(s) + Akp s(s + 1= )c(s) +

s2 Ac(s);

kD

s2 Ac(s);

b = (s + 1= )c(s)
AkD
iii. K =
;
a = s(s + 1= )d(s) + Akp s(s + 1= )c(s) + AkI (s + 1= )c(s);
b = s2 c(s)
iv. K = 1= ;
a = s2 d(s) + Akp s2 c(s) + AkI sc(s);
b = sd(s) + Akp sc(s) + AkI c(s) + AkD s2 c(s)

2015 Pearson Education, Inc., Upper Saddle River, NJ. All rights reserved. This publication is protected by Copyright and written permission should be obtained
from the publisher prior to any prohibited reproduction, storage in a retrieval system, or transmission in any form or by any means, electronic, mechanical, photocopying,
recording, or likewise. For information regarding permission(s), write to: Rights and Permissions Department, Pearson Education, Inc., Upper Saddle River, NJ 07458.

5003

Problems and solutions for Section 5.2


2. Roughly sketch the root loci for the pole-zero maps as shown in Fig. 5.44
without the aid of a computer. Show your estimates of the center and
angles of the asymptotes, a rough evaluation of arrival and departure
angles for complex poles and zeros, and the loci for positive values of the
parameter K. Each pole-zero map is from a characteristic equation of the
form
b(s)
= 0;
1+K
a(s)
where the roots of the numerator b(s) are shown as small circles o and the
roots of the denominator a(s) are shown as 0 s on the s-plane. Note that
in Fig. 5.44(c) there are two poles at the origin.
Solution:
We had to make up some numbers to do it on Matlab, so the results
partly depend on what was dreamed up, but the idea here is just get the
basic rules right.
(a) a(s) = s2 + s; b(s) = s + 2
Breakin(s): -3.414
Breakaway(s): -0.586
(b) a(s) = s2 + 0:2s + 1; b(s) = s + 1
Angle of departure: 137:9
Breakin(s): -2.342
(c) a(s) = s2 ; b(s) = (s + 1)
Breakin(s): -2
(d) a(s) = s2 + s; b(s) = s2 + 5s + 6
Breakin(s): -2.366
Breakaway(s): -0.634
(e) a(s) = s3 + 3s2 + 4s 8
Center of asymptotes: -1
Angles of asymptotes:
60 ; 180
Angle of departure:
56:3
(f) a(s) = s5 + 3s4 + s3 5s2 0:5; b(s) = s + 1
Center of asymptotes: -0.5
Angles of asymptotes:
45 ; 135
Angle of departure:
115:8 at s = 2:01 + 1:01j
70:5 at s = 0:01 + 0:31j
Breakin(s): 0.05
Breakaway(s): 0:652

2015 Pearson Education, Inc., Upper Saddle River, NJ. All rights reserved. This publication is protected by Copyright and written permission should be obtained
from the publisher prior to any prohibited reproduction, storage in a retrieval system, or transmission in any form or by any means, electronic, mechanical, photocopying,
recording, or likewise. For information regarding permission(s), write to: Rights and Permissions Department, Pearson Education, Inc., Upper Saddle River, NJ 07458.

5004

CHAPTER 5. THE ROOT-LOCUS DESIGN METHOD

Root loci for Problem 5.2


2

Imaginary Axis

Imaginary Axis

2
1
0
-1
-2
-10

1
0
-1

-5

-2
-6

-4

-2
Real Axis

Real Axis

1
0
-1
-2
-4

0.5
0
-0.5

-2

-1
-4

-2

Real Axis

Imaginary Axis

Imaginary Axis

0
Real Axis

10
5
0
-5
-10
-10

Imaginary Axis

Imaginary Axis

2
0
-2

-5

0
Real Axis

-4
-4

-2

0
Real Axis

2015 Pearson Education, Inc., Upper Saddle River, NJ. All rights reserved. This publication is protected by Copyright and written permission should be obtained
from the publisher prior to any prohibited reproduction, storage in a retrieval system, or transmission in any form or by any means, electronic, mechanical, photocopying,
recording, or likewise. For information regarding permission(s), write to: Rights and Permissions Department, Pearson Education, Inc., Upper Saddle River, NJ 07458.

5005
3. For the characteristic equation
1+

K
= 0;
s2 (s + 1)(s + 5)

(a) Draw the real-axis segments of the corresponding root locus.


(b) Sketch the asymptotes of the locus for K ! 1.
(c) Sketch the locus.

(d) Verify your sketch with a Matlab plot.

Solution:
(a) The real axis segment is
(b)

6=4 =

1:5;

1>
45 ;

>

5.

135

(c) The plot is shown below.

Root Locus

Imaginary Axis

-2

-4

-6

-8

-6

-4

-2
Real Axis

Solution for Problem 5.3

2015 Pearson Education, Inc., Upper Saddle River, NJ. All rights reserved. This publication is protected by Copyright and written permission should be obtained
from the publisher prior to any prohibited reproduction, storage in a retrieval system, or transmission in any form or by any means, electronic, mechanical, photocopying,
recording, or likewise. For information regarding permission(s), write to: Rights and Permissions Department, Pearson Education, Inc., Upper Saddle River, NJ 07458.

5006

CHAPTER 5. THE ROOT-LOCUS DESIGN METHOD

4. Real poles and zeros. Sketch the root locus with respect to K for the
equation 1+KL(s) = 0 and the listed choices for L(s). Be sure to give the
asymptotes and the arrival and departure angles at any complex zero or
pole. After completing each hand sketch, verify your results using Matlab.
Turn in your hand sketches and the Matlab results on the same scales.

(a) L(s) =

2
s(s + 1)(s + 5)(s + 10)

(b) L(s) =

(s + 3)
s(s + 1)(s + 5)(s + 10)

(c) L(s) =

(s + 2)(s + 4)
s(s + 1)(s + 5)(s + 10)

(d) L(s) =

(s + 2)(s + 6)
s(s + 1)(s + 5)(s + 10)

Solution:
All the root locus plots are displayed at the end of the solution set for this
problem.

(a)

4;

45 ;

(b)

4:33;

(c)

5;

90

(d)

4;

90

135

60 ; 180

2015 Pearson Education, Inc., Upper Saddle River, NJ. All rights reserved. This publication is protected by Copyright and written permission should be obtained
from the publisher prior to any prohibited reproduction, storage in a retrieval system, or transmission in any form or by any means, electronic, mechanical, photocopying,
recording, or likewise. For information regarding permission(s), write to: Rights and Permissions Department, Pearson Education, Inc., Upper Saddle River, NJ 07458.

5007

b
15

10

10

Imaginary Axis

Imaginary Axis

a
15

-5

-5

-10

-10

-15
-15

-10

-5
Real Axis

-15
-15

-10

15

10

10

-5

-10

-10

-10

-5
Real Axis

-5

-15
-15

15

Imaginary Axis

Imaginary Axis

-5
Real Axis

-15
-15

-10

-5
Real Axis

Root loci for Problem 5.4

2015 Pearson Education, Inc., Upper Saddle River, NJ. All rights reserved. This publication is protected by Copyright and written permission should be obtained
from the publisher prior to any prohibited reproduction, storage in a retrieval system, or transmission in any form or by any means, electronic, mechanical, photocopying,
recording, or likewise. For information regarding permission(s), write to: Rights and Permissions Department, Pearson Education, Inc., Upper Saddle River, NJ 07458.

5008

CHAPTER 5. THE ROOT-LOCUS DESIGN METHOD

5. Complex poles and zeros. Sketch the root locus with respect to K for the
equation 1+KL(s) = 0 and the listed choices for L(s). Be sure to give the
asymptotes and the arrival and departure angles at any complex zero or
pole. After completing each hand sketch, verify your results using Matlab.
Turn in your hand sketches and the Matlab results on the same scales.

(a) L(s) =

1
s2 + 3s + 10

(b) L(s) =

1
s(s2 + 3s + 10)

(c) L(s) =

(s2 + 2s + 8)
s(s2 + 2s + 10)

(d) L(s) =

(s2 + 2s + 12)
s(s2 + 2s + 10)

(e) L(s) =

(s2 + 1)
s(s2 + 4)

(f) L(s) =

(s2 + 4)
s(s2 + 1)

Solution:
All the root locus plots are displayed at the end of the solution set for this
problem.

(a)

1:5;

(b)

1;

(c)

i
i

90 ;

= 90 at s =

60 ; 180 ;

1:5 + 2:78j

28:3 at s =

= 0; i = 180 ; d = 161:6 at s =
= 200:7 at s = 1 + 2:65j

1 + 3j;

= 0; i = 180 ;
= 16:8 at s =

1 + 3j;

1:5 + 2:78j

(d)

= 18:4 at s =
1 + 3:32j

(e)

= 0;

= 180 ;

= 180 at s = 2j;

(f)

= 0;

= 180 ;

= 0 at s = j;

= 180 at s = j

= 0 at s = 2j

2015 Pearson Education, Inc., Upper Saddle River, NJ. All rights reserved. This publication is protected by Copyright and written permission should be obtained
from the publisher prior to any prohibited reproduction, storage in a retrieval system, or transmission in any form or by any means, electronic, mechanical, photocopying,
recording, or likewise. For information regarding permission(s), write to: Rights and Permissions Department, Pearson Education, Inc., Upper Saddle River, NJ 07458.

5009

b
20
Imaginary Axis

Imaginary Axis

20
10
0
-10
-20
-3

-2

-1
Real Axis

10
0
-10
-20
-20

-10
0
Real Axis

d
4
Imaginary Axis

Imaginary Axis

4
2
0
-2
-4
-3

-2

-1
Real Axis

2
0
-2
-4
-3

-2

0.5

4
Imaginary Axis

Imaginary Axis

-1
Real Axis
f

4
2
0
-2
-4
-2

10

-1

Real Axis

2
0
-2
-4
-1.5

-1

-0.5
Real Axis

Root loci for Problem 5.5

2015 Pearson Education, Inc., Upper Saddle River, NJ. All rights reserved. This publication is protected by Copyright and written permission should be obtained
from the publisher prior to any prohibited reproduction, storage in a retrieval system, or transmission in any form or by any means, electronic, mechanical, photocopying,
recording, or likewise. For information regarding permission(s), write to: Rights and Permissions Department, Pearson Education, Inc., Upper Saddle River, NJ 07458.

5010

CHAPTER 5. THE ROOT-LOCUS DESIGN METHOD

6. Multiple poles at the origin. Sketch the root locus with respect to K for
the equation 1 + KL(s) = 0 and the listed choices for L(s). Be sure to
give the asymptotes and the arrival and departure angles at any complex
zero or pole. After completing each hand sketch, verify your results using
Matlab. Turn in your hand sketches and the Matlab results on the same
scales.

(a) L(s) =

1
s2 (s + 8)

(b) L(s) =

1
s3 (s + 8)

(c) L(s) =

1
s4 (s + 8)

(d) L(s) =

(s + 3)
s2 (s + 8)

(e) L(s) =

(s + 3)
s3 (s + 4)

(f) L(s) =

(s + 1)2
s3 (s + 4)

(g) L(s) =

(s + 1)2
s3 (s + 10)2

Solution:
All the root locus plots are displayed at the end of the solution set for this
problem.
(a)

2:67;

(b)

2;

(c)

1:6;

36 ;

(d)

2:5;

90

(e)

0:33;

(f)

3;

90

(g)

6;

60 ; 180

60 ; 180

45 ;

135
108 ; 180

60 ; 180

2015 Pearson Education, Inc., Upper Saddle River, NJ. All rights reserved. This publication is protected by Copyright and written permission should be obtained
from the publisher prior to any prohibited reproduction, storage in a retrieval system, or transmission in any form or by any means, electronic, mechanical, photocopying,
recording, or likewise. For information regarding permission(s), write to: Rights and Permissions Department, Pearson Education, Inc., Upper Saddle River, NJ 07458.

5011

b
Imaginary Axis

Imaginary Axis

a
20
10
0
-10
-20
-15

-10

-5

0
Real Axis

5
0
-5
-10

10

-10

-5

Imaginary Axis

Imaginary Axis

Real Axis

10
5
0

50

-5
-15

-10

-5
Real Axis

-50
-10

10

-5

Imaginary Axis

Imaginary Axis

Real Axis

0.5
0
-0.5

10
5
0
-5
-10

-1

-5

-4

-3

-2
-1
Real Axis

-5
Real Axis

Imaginary Axis

-5

-4

-3

-2
Real Axis

-1

5
0
-5
-10

-10

Solution for Problem 5.6

2015 Pearson Education, Inc., Upper Saddle River, NJ. All rights reserved. This publication is protected by Copyright and written permission should be obtained
from the publisher prior to any prohibited reproduction, storage in a retrieval system, or transmission in any form or by any means, electronic, mechanical, photocopying,
recording, or likewise. For information regarding permission(s), write to: Rights and Permissions Department, Pearson Education, Inc., Upper Saddle River, NJ 07458.

5012

CHAPTER 5. THE ROOT-LOCUS DESIGN METHOD

7. Mixed real and complex poles. Sketch the root locus with respect to K for
the equation 1 + KL(s) = 0 and the listed choices for L(s). Be sure to
give the asymptotes and the arrival and departure angles at any complex
zero or pole. After completing each hand sketch, verify your results using
Matlab. Turn in your hand sketches and the Matlab results on the same
scales.

(a) L(s) =

(s + 3)
s(s + 10)(s2 + 2s + 2)

(b) L(s) =

(s + 3)
s2 (s + 10)(s2 + 6s + 25)

(c) L(s) =

s2 (s

(s + 3)2
+ 10)(s2 + 6s + 25)

(d) L(s) =

(s + 3)(s2 + 4s + 68)
s2 (s + 10)(s2 + 4s + 85)

(e) L(s) =

[(s + 1)2 + 1]
s2 (s + 2)(s + 3)

Solution:
All the plots are attached at the end of the solution set.

(a)

3;

(b)

3:25;

45 ;

(c)

3:33;

60 ; 180 ;

(d)

60 ; 180 ;

135 ;

24:8 at s =
d

1+j

103:5 at s =
13:5 at s =

= 3:5; i = 90 ; d = 100:2 at s =
= 99:8 at s = 2 + 8j

3 + 4j

3 + 4j

2 + 9j;

(e)

1:5;

90 ;

= 71:6 at s =

1+j

2015 Pearson Education, Inc., Upper Saddle River, NJ. All rights reserved. This publication is protected by Copyright and written permission should be obtained
from the publisher prior to any prohibited reproduction, storage in a retrieval system, or transmission in any form or by any means, electronic, mechanical, photocopying,
recording, or likewise. For information regarding permission(s), write to: Rights and Permissions Department, Pearson Education, Inc., Upper Saddle River, NJ 07458.

5013

c
10

10

1
0

Imaginary Axis

Imaginary Axis

Imaginary Axis

2
0
-2

-1

-8

-8

-10

-10
-10

-5
Real Axis

0
-2

-6

-6

-3

-4

-4
-2

-15

-10

-5
Real Axis

-15

-10

-5
Real Axis

e
5
4

15

3
10
Imaginary Axis

Imaginary Axis

2
5

1
0
-1

-5
-2
-10

-3
-4

-15

-5
-15

-10

-5
Real Axis

-3

-2
-1
Real Axis

Solution for Problem 5.7

2015 Pearson Education, Inc., Upper Saddle River, NJ. All rights reserved. This publication is protected by Copyright and written permission should be obtained
from the publisher prior to any prohibited reproduction, storage in a retrieval system, or transmission in any form or by any means, electronic, mechanical, photocopying,
recording, or likewise. For information regarding permission(s), write to: Rights and Permissions Department, Pearson Education, Inc., Upper Saddle River, NJ 07458.

5014

CHAPTER 5. THE ROOT-LOCUS DESIGN METHOD

8. RHP and zeros. Sketch the root locus with respect to K for the equation 1 + KL(s) = 0 and the listed choices for L(s). Be sure to give the
asymptotes and the arrival and departure angles at any complex zero or
pole. After completing each hand sketch, verify your results using Matlab.
Turn in your hand sketches and the Matlab results on the same scales.
1
s+2
; the model for a case of magnetic levitation
s + 10 s2 1
with lead compensation.

(a) L(s) =

s+2
1
; the magnetic levitation system with inte2
s(s + 10) (s
1)
gral control and lead compensation.

(b) L(s) =

(c) L(s) =

1
s2

s2 + 2s + 1
: What is the largest value that can
s(s + 20)2 (s2 2s + 2)
be obtained for the damping ratio of the stable complex roots on this
locus?

(d) L(s) =

(e) L(s) =
(f) L(s) =

(s + 2)
;
s(s 1)(s + 6)2
1
1)[(s + 2)2 + 3]

(s

Solution:

(a)

4;

(b)

2:67;

(c)

1;

(d)

90
=

60 ; 180

= 180

= 12; i = 60 ; 180 ; d = 92:7 at s = 1 + j


The maximum damping ratio is obtained at a point at the smallest
angle o the horizontal. From the locus, the maximum damping is
0:31 when K 2600.

(e)

3;

60 ; 180

(f)

1;

60 ; 180 ;

60:0 at s =

2 + 1:73j

2015 Pearson Education, Inc., Upper Saddle River, NJ. All rights reserved. This publication is protected by Copyright and written permission should be obtained
from the publisher prior to any prohibited reproduction, storage in a retrieval system, or transmission in any form or by any means, electronic, mechanical, photocopying,
recording, or likewise. For information regarding permission(s), write to: Rights and Permissions Department, Pearson Education, Inc., Upper Saddle River, NJ 07458.

5015
10

10

plot a

-5

0.5
Imag Axis

5
Imag Axis

Imag Axis

-10

plot c

plot b

-5

-10

-10

-5
0
Real Axis

plot d

-0.5

-10

-1
-1

-5
0
Real Axis

10

10

plot f

Imag Axis

0
-5
-10

2
Imag Axis

4
plot e

15

Imag Axis

0
1
Real Axis

-5

-2

-15
-20
-5

0
Real Axis

-10
-8

-6

-4 -2
0
Real Axis

-4

-2

0
Real Axis

Solution for Problem 5.8

2015 Pearson Education, Inc., Upper Saddle River, NJ. All rights reserved. This publication is protected by Copyright and written permission should be obtained
from the publisher prior to any prohibited reproduction, storage in a retrieval system, or transmission in any form or by any means, electronic, mechanical, photocopying,
recording, or likewise. For information regarding permission(s), write to: Rights and Permissions Department, Pearson Education, Inc., Upper Saddle River, NJ 07458.

5016

CHAPTER 5. THE ROOT-LOCUS DESIGN METHOD

9. Put the characteristic equation of the system shown in Fig. 5.45 in root
locus form with respect to the parameter , and identify the corresponding
L(s); a(s); and b(s): Sketch the root locus with respect to the parameter
, estimate the closed-loop pole locations, and sketch the corresponding
step responses when = 0; 0:5, and 2. Use Matlab to check the accuracy
of your approximate step responses.

Figure 5.45: Control system for Problem 5.9


Solution:
The characteristic equation is s2 +2s+5+5 s = 0 and L(s) =
the root locus and step responses are plotted below.

s2

s
:
+ 2s + 5

root locus

StepResponse

2.5

1.4

a l p h a = 0

1.2
1.5

a l p h a = 0 . 5
a l p h a = 2

0.5
Amplitude

Imaginary Axis

0.8

0.6
-0.5

-1

0.4

-1.5
0.2
-2

-2.5

-6

-5

-4

-3

-2
Real Axis

-1

10
Time(sec)

15

2015 Pearson Education, Inc., Upper Saddle River, NJ. All rights reserved. This publication is protected by Copyright and written permission should be obtained
from the publisher prior to any prohibited reproduction, storage in a retrieval system, or transmission in any form or by any means, electronic, mechanical, photocopying,
recording, or likewise. For information regarding permission(s), write to: Rights and Permissions Department, Pearson Education, Inc., Upper Saddle River, NJ 07458.

20

5017
10. Use the Matlab function rltool to study the behavior of the root locus of
1 + KL(s) for
(s + a)
L(s) =
s(s + 1)(s2 + 8s + 52)
as the parameter a is varied from 0 to 10, paying particular attention to
the region between 2:5 and 3:5. Verify that a multiple root occurs at a
complex value of s for some value of a in this range.
Solution:
For small values of ; the locus branch from 0; 1 makes a circular path
around the zero and the branches from the complex roots curve o toward
the asymptotes. For large values of
the branches from the complex
roots break into the real axis and those from 0; 1 curve o toward the
asymptotes. At about = 3:11 these loci touch corresponding to complex
multiple roots.

a = 2 .5

Imaginary Axis

10

-2

-2

-2

-4

-4

-4

-6

-6

-6

-8

-8

-1 0
-1 4

-1 2

-1 0

-8

-6

-4

-2

-1 0
-1 4

-8

-1 2

-1 0

-8

-6

R e a l A x is

-4

-2

-1 0
-1 4

Imaginary Axis

Imaginary Axis

-2

-2

-4

-4

-4

-6

-6

-6

-8

-8

-6

-4

-2

-1 0
-1 4

-4

-2

-2

-2

-8

-6

a=10
10

-1 0

-8

a=5
10

-1 2

-1 0

R e a l A x is

10

-1 0
-1 4

-1 2

R e a l A x is

a = 3 .5

Imaginary Axis

a=3

10

Imaginary Axis

Imaginary Axis

a=0
10

-8

-1 2

-1 0

-8

R e a l A x is

-6

-4
R e a l A x is

-2

-1 0
-1 4

-1 2

-1 0

-8

-6

-4
R e a l A x is

Solution for Problem 5.10

2015 Pearson Education, Inc., Upper Saddle River, NJ. All rights reserved. This publication is protected by Copyright and written permission should be obtained
from the publisher prior to any prohibited reproduction, storage in a retrieval system, or transmission in any form or by any means, electronic, mechanical, photocopying,
recording, or likewise. For information regarding permission(s), write to: Rights and Permissions Department, Pearson Education, Inc., Upper Saddle River, NJ 07458.

5018

CHAPTER 5. THE ROOT-LOCUS DESIGN METHOD

11. Use Rouths criterion to nd the range of the gain K for which the systems
in Fig. 5.46 are stable, and use the root locus to conrm your calculations.

Figure 5.46: Feedback systems for Problem 5.11

Solution:

(a) The system is stable for 0


K
478:234. The root locus of the
system and the location of the roots at the crossover points are shown
in the left plot.
(b) There is a pole in the right hand plane thus the system is unstable
for all values of K as shown in the right plot.

2015 Pearson Education, Inc., Upper Saddle River, NJ. All rights reserved. This publication is protected by Copyright and written permission should be obtained
from the publisher prior to any prohibited reproduction, storage in a retrieval system, or transmission in any form or by any means, electronic, mechanical, photocopying,
recording, or likewise. For information regarding permission(s), write to: Rights and Permissions Department, Pearson Education, Inc., Upper Saddle River, NJ 07458.

5019
Root Locus

plot a
Imag Axis

Imag Axis

Root Locus
5

plot b

-5
-5
-10

-5
Real Axis

-3

-2.5

-2

-1

-0.5

-1

-0.5

Root Locus
4

plot c
Imag Axis

Imag Axis

Root Locus
1.5
1

-1.5
Real Axis

0.5
0
-0.5

plot d

2
0
-2

-1
-1.5
-6

-4
Real Axis

-2

-4

-3

-2.5

-2

-1.5
Real Axis

Solution for Problem 5.11

2015 Pearson Education, Inc., Upper Saddle River, NJ. All rights reserved. This publication is protected by Copyright and written permission should be obtained
from the publisher prior to any prohibited reproduction, storage in a retrieval system, or transmission in any form or by any means, electronic, mechanical, photocopying,
recording, or likewise. For information regarding permission(s), write to: Rights and Permissions Department, Pearson Education, Inc., Upper Saddle River, NJ 07458.

5020

CHAPTER 5. THE ROOT-LOCUS DESIGN METHOD

12. Sketch the root locus for the characteristic equation of the system for
which
(s + 2)
L(s) = 2
;
s (s + 5)
and determine the value of the root-locus gain for which the complex conjugate poles have the maximum damping ratio. What is the approximate
value of the damping?
Solution:
Plot the system on Matlab using rlocus(sys), and use [K]= rlocnd(sys)
to pick the gain for the maximum damping. We nd that the maximum
damping, = 0.275 when K 10:7.

R oot Loc us
15
0 .3 8

0 .2 6

0 .1 9

0 .1 3

0 .0 8 5

0 .0 4

14

12

10

10
0 .5 2

6
5
0 .8

Imaginar y Axis

0 .8
-5

-10

0 .5 2
10

12

0 .2 6

0 .3 8
-15
-6

-5

-4

0 .1 9
-3

0 .1 3
-2

0 .0 8 5
-1

0 .0 4

14
0

R e a l Ax is

Root locus with 0.275 damping marked

2015 Pearson Education, Inc., Upper Saddle River, NJ. All rights reserved. This publication is protected by Copyright and written permission should be obtained
from the publisher prior to any prohibited reproduction, storage in a retrieval system, or transmission in any form or by any means, electronic, mechanical, photocopying,
recording, or likewise. For information regarding permission(s), write to: Rights and Permissions Department, Pearson Education, Inc., Upper Saddle River, NJ 07458.

5021
13. For the system in Fig. 5.47,

Figure 5.47: Feedback system for Problem 5.13

(a) Find the locus of closed-loop roots with respect to K.


(b) Is there a value of K that will cause all roots to have a damping ratio
greater than 0:5?
(c) Find the values of K that yield closed-loop poles with the damping
ratio = 0:707.
(d) Use Matlab to plot the response of the resulting design to a reference
step.
Solution:
(a) The locus is plotted below
(b) There is a K which will make the dominant poles have damping
0.5 but none that will make the poles from the resonance have that
much damping.
(c) Using rlocnd, the gain is about 35.
(d) The step response shows the basic form of a well damped response
with the vibration of the response element added.

0.84

8 0.92
6

RootLocus
0.74
0.6

1.2
0.965

1
0.99

Amp litu d e

Imag Axis

StepResponse

1.4

0.42 0.22

2
200 17.5

15 12.5

10

7.5

2.5

-2

0.8
0.6

0.99
0.4

-4
-6
-8
-20

0.965
0.2
0.92

0.84
-15

0.74
0.6 0.42 0.22
-10
-5
RealAxis

3
Time(sec)

2015 Pearson Education, Inc., Upper Saddle River, NJ. All rights reserved. This publication is protected by Copyright and written permission should be obtained
from the publisher prior to any prohibited reproduction, storage in a retrieval system, or transmission in any form or by any means, electronic, mechanical, photocopying,
recording, or likewise. For information regarding permission(s), write to: Rights and Permissions Department, Pearson Education, Inc., Upper Saddle River, NJ 07458.

5022

CHAPTER 5. THE ROOT-LOCUS DESIGN METHOD


Root locus and step response for Problem 5.13

2015 Pearson Education, Inc., Upper Saddle River, NJ. All rights reserved. This publication is protected by Copyright and written permission should be obtained
from the publisher prior to any prohibited reproduction, storage in a retrieval system, or transmission in any form or by any means, electronic, mechanical, photocopying,
recording, or likewise. For information regarding permission(s), write to: Rights and Permissions Department, Pearson Education, Inc., Upper Saddle River, NJ 07458.

5023
14. For the feedback system shown in Fig. 5.48, nd the value of the gain K
that results in dominant closed-loop poles with a damping ratio = 0:5.

Figure 5.48: Feedback system for Problem 5.14

Solution:

Use block diagram reduction to nd the characteristic equation of the


closed loop system, then divide that up into terms with and without K to
10s
nd the root locus form, where L(s) = 2
: Plugging into Matlab
s + s + 10
and using rlocnd produces the required gain to be K = 0:22:The locus is

2015 Pearson Education, Inc., Upper Saddle River, NJ. All rights reserved. This publication is protected by Copyright and written permission should be obtained
from the publisher prior to any prohibited reproduction, storage in a retrieval system, or transmission in any form or by any means, electronic, mechanical, photocopying,
recording, or likewise. For information regarding permission(s), write to: Rights and Permissions Department, Pearson Education, Inc., Upper Saddle River, NJ 07458.

5024

CHAPTER 5. THE ROOT-LOCUS DESIGN METHOD

R oot Locus
4
0.84

0.72

0.58

0.44

0.3

0.14

3
0.92

0.98

Imaginary Axis

6
0

-1
0.98

-2

0.92
-3

0.72

0.84
-4
-6

-5

-4

0.58
-3

0.44
-2

0.3
-1

0.14
0

R eal Axis

Root locus with 0.5 damping marked

2015 Pearson Education, Inc., Upper Saddle River, NJ. All rights reserved. This publication is protected by Copyright and written permission should be obtained
from the publisher prior to any prohibited reproduction, storage in a retrieval system, or transmission in any form or by any means, electronic, mechanical, photocopying,
recording, or likewise. For information regarding permission(s), write to: Rights and Permissions Department, Pearson Education, Inc., Upper Saddle River, NJ 07458.

5025

Problems and solutions for Section 5.3


15. A simplied model of the longitudinal motion of a certain helicopter near
hover has the transfer function
G(s) =

9:8(s2 0:5s + 6:3)


:
(s + 0:66)(s2 0:24s + 0:15)

and the characteristic equation 1 + Dc (s)G(s) = 0. Let Dc (s) = kp at


rst.
(a) Compute the departure and arrival angles at the complex poles and
zeros.
(b) Sketch the root locus for this system for parameter K = 9:8kp . Use
axes -4 x 4; 3 y 3;
(c) Verify your answer using Matlab. Use the command axis([-4 4 -3 3])
to get the right scales.
(d) Suggest a practical (at least as many poles as zeros) alternative compensation Dc (s) which will at least result in a stable system.

Solution:
(a)

d
a

=
=

180
25:26
90 + 266:5 + 92:6 = 63:83 ;
90 + 86:5 + 69:9 + 87:4
180 = 26:11

(b) (c)
Root Locus
3

Imaginary Axis

-1

-2

-3
-4

-3

-2

-1

Real A xis

Problem 5.15(b)(c)

2015 Pearson Education, Inc., Upper Saddle River, NJ. All rights reserved. This publication is protected by Copyright and written permission should be obtained
from the publisher prior to any prohibited reproduction, storage in a retrieval system, or transmission in any form or by any means, electronic, mechanical, photocopying,
recording, or likewise. For information regarding permission(s), write to: Rights and Permissions Department, Pearson Education, Inc., Upper Saddle River, NJ 07458.

5026

CHAPTER 5. THE ROOT-LOCUS DESIGN METHOD


(d) For this problem a double lead is needed to bring the roots into the
left half-plane. The plot shows the rootlocus for control for. Let
(s + :66)(s + :33)
:
Dc (s) =
(s + 5)2
Root Loc us
3

Imaginary Axis

-1

-2

-3
-4

-3

-2

-1

Real A xis

Problem 5.15(d)

2015 Pearson Education, Inc., Upper Saddle River, NJ. All rights reserved. This publication is protected by Copyright and written permission should be obtained
from the publisher prior to any prohibited reproduction, storage in a retrieval system, or transmission in any form or by any means, electronic, mechanical, photocopying,
recording, or likewise. For information regarding permission(s), write to: Rights and Permissions Department, Pearson Education, Inc., Upper Saddle River, NJ 07458.

5027

Figure 5.49: Control system for Problem 5.16

16. For the system given in Fig. 5.49,


(a) Plot the root locus of the characteristic equation as the parameter
K1 is varied from 0 to 1 with = 2. Give the corresponding L(s);
a(s); and b(s):
(b) Repeat part (a) with
value?

= 5. Is there anything special about this

(c) Repeat part (a) for xed K1 = 2 with the parameter K =


from 0 to 1.

varying

Solution:
Use block diagram reduction to nd the characteristic equation of the
closed-loop system:
1+

10K1
s(s + )
0:1
+ 0:2s + 1
s(s + 10)(s + )
K1
or
s(s + )(s + 11) + 2K1 (s + 5) = 0

=0

The root locus for each part is attached at the end.


(a) Substituting = 2 and divide the equation above up into terms with
and without K1 to nd Evans form:
1 + K1

2(s + 5)
=0
s(s + 2)(s + 11)

L(s) =

2(s + 5)
s(s + 2)(s + 11)

(b) Substituting
= 5 and rewrite the equation in Evans form with
respect to K1 :
1 + K1

2(s + 5)
=0
s(s + 5)(s + 11)

We have a pole-zero cancellation at s =

L(s) =

2
s(s + 11)

5.

2015 Pearson Education, Inc., Upper Saddle River, NJ. All rights reserved. This publication is protected by Copyright and written permission should be obtained
from the publisher prior to any prohibited reproduction, storage in a retrieval system, or transmission in any form or by any means, electronic, mechanical, photocopying,
recording, or likewise. For information regarding permission(s), write to: Rights and Permissions Department, Pearson Education, Inc., Upper Saddle River, NJ 07458.

5028

CHAPTER 5. THE ROOT-LOCUS DESIGN METHOD


(c) Substituting K1 = 2 and divide the characteristic equation up into
terms with and without to nd Evans form:
1+

s(s + 11)
=0
s2 (s + 11) + 4(s + 5)

L(s) =

s(s + 11)
s3 + 11s2 + 4s + 20

plot a

plot b

40

30

1.5
1

Imaginary Axis

Imaginary Axis

20
10
0
-10

0.5
0
-0.5

-20

-1

-30

-1.5

-40
-15

-10

-5
Real Axis

-2
-40

-30

-20

-10
Real Axis

10

20

plot c
2
1.5

Imaginary Axis

1
0.5
0
-0.5
-1
-1.5
-2
-30

-25

-20

-15

-10
Real Axis

-5

10

Solution for Problem 5.16

2015 Pearson Education, Inc., Upper Saddle River, NJ. All rights reserved. This publication is protected by Copyright and written permission should be obtained
from the publisher prior to any prohibited reproduction, storage in a retrieval system, or transmission in any form or by any means, electronic, mechanical, photocopying,
recording, or likewise. For information regarding permission(s), write to: Rights and Permissions Department, Pearson Education, Inc., Upper Saddle River, NJ 07458.

5029

Figure 5.50: Control system for Problem 5.17

17. For the system shown in Fig. 5.50, determine the characteristic equation
and sketch the root locus of it with respect to positive values of the parameter c. Give L(s), a(s); and b(s) and be sure to show with arrows the
direction in which c increases on the locus.
Solution:
L(s) =

a(s)
s2 + 9
=
+ 144s
b(s)

s3

R oot Loc us

10

Imag Axis

-5

-10

-18

-16

-14

-12

-10

-8

-6

-4

-2

R eal Ax is

Solution for Problem 5.17

2015 Pearson Education, Inc., Upper Saddle River, NJ. All rights reserved. This publication is protected by Copyright and written permission should be obtained
from the publisher prior to any prohibited reproduction, storage in a retrieval system, or transmission in any form or by any means, electronic, mechanical, photocopying,
recording, or likewise. For information regarding permission(s), write to: Rights and Permissions Department, Pearson Education, Inc., Upper Saddle River, NJ 07458.

5030

CHAPTER 5. THE ROOT-LOCUS DESIGN METHOD

18. Suppose you are given a system with the transfer function
L(s) =

(s + z)
;
(s + p)2

where z and p are real and z > p. Show that the root locus for 1+KL(s) =
0 with respect to K is a circle centered at z with radius given by
r = (z

p):

Hint: Assume s + z = rej and show that L(s) is real and negative for
real under this assumption.
Solution:
Assume s + z = (z

p)ej . Then L(s) can be written as:

L =
=
=

(z

p)ej
z + p)
j
(z p)e
(z p)2 (ej
1)2
1
((z

(z
=

p)ej

p)( 4)

1
4(z

ej

=2

e
2j

j =2

1
p) (sin( =2))2

Because z > p; this function is real and negative for real


these points are on the locus.

and therefore

2015 Pearson Education, Inc., Upper Saddle River, NJ. All rights reserved. This publication is protected by Copyright and written permission should be obtained
from the publisher prior to any prohibited reproduction, storage in a retrieval system, or transmission in any form or by any means, electronic, mechanical, photocopying,
recording, or likewise. For information regarding permission(s), write to: Rights and Permissions Department, Pearson Education, Inc., Upper Saddle River, NJ 07458.

5031
19. The loop transmission of a system has two poles at s = 1 and a zero
at s = 2. There is a third real-axis pole p located somewhere to the
left of the zero. Several dierent root loci are possible, depending on the
exact location of the third pole. The extreme cases occur when the pole
is located at innity or when it is located at s = 2. Give values for p
and sketch the three distinct types of loci.

Solution:

Root Locus

Root Locus
0.5

20

pole is at negativ e inf inity

pole is at s=-2
Imag Axis

Imag Axis

10
0

-10
-20
-0.5
-100

-80

-60

-40

-20

-2

Real Axis

-1.5

-1

-0.5

Real Axis

Root Locus
5

Imag Axis

pole is slightly to the lef t of zero

-5
-3

-2.5

-2

-1.5

-1

-0.5

Real Axis

Solution for Problem 5.19

2015 Pearson Education, Inc., Upper Saddle River, NJ. All rights reserved. This publication is protected by Copyright and written permission should be obtained
from the publisher prior to any prohibited reproduction, storage in a retrieval system, or transmission in any form or by any means, electronic, mechanical, photocopying,
recording, or likewise. For information regarding permission(s), write to: Rights and Permissions Department, Pearson Education, Inc., Upper Saddle River, NJ 07458.

5032

CHAPTER 5. THE ROOT-LOCUS DESIGN METHOD

20. For the feedback conguration of Fig. 5.51, use asymptotes, center of
asymptotes, angles of departure and arrival, and the Routh array to sketch
root loci for the characteristic equations of the listed feedback control
systems versus the parameter K: Use Matlab to verify your results.
K
;
s(s + 1 + 3j)(s + 1 3j)
K
(b) G(s) = 2 ;
s
K(s + 5)
(c) G(s) =
;
(s + 1)
K(s + 3 + 4j)(s + 3 4j)
(d) G(s) =
;
s(s + 1 + 2j)(s + 1 2j)
(a) G(s) =

s+2
s+8
s+1
H(s) =
s+3
s+7
H(s) =
s+3
H(s) =

H(s) = 1 + 3s

Figure 5.51: Feedback system for Problem 5.20


Solution:
The root locus for each part is attached at the end.
(a)
L(s)
-

(s + 2)
s(s + 1 + 3j)(s + 1

Asymptotes: 4 1 = 3
Center of asymptotes: = 2:67
Angle of asymptotoes: = 60 ; 180
Angle of departure: d = 29:93 at s =
Imaginary-axis crossings:
(s)

3j)(s + 8)

1 + 3j

= s4 + 10s3 + 26s2 + (80 + K) s + 2K


s4 :
1
26
2K
s3 :
10
80 + K
K
s2 :
18 10
2K
2
K
100K+14400
s:
180 K
s0 :
2K

Rouths test gives 0 < K < 80 for stability. Solving


K = 80, the crossings are s = 4j.

(s) with

2015 Pearson Education, Inc., Upper Saddle River, NJ. All rights reserved. This publication is protected by Copyright and written permission should be obtained
from the publisher prior to any prohibited reproduction, storage in a retrieval system, or transmission in any form or by any means, electronic, mechanical, photocopying,
recording, or likewise. For information regarding permission(s), write to: Rights and Permissions Department, Pearson Education, Inc., Upper Saddle River, NJ 07458.

5033
(b)
L(s)
- Asymptotes: 3

(s + 1)
s2 (s + 3)

1=2

- Center of asymptotes:

- Angle of asymptotoes:

90

- Imaginary-axis crossings:
(s)

= s3 + 3s2 + Ks + K
s3 : 1 K
s2 : 3 K
s : 2K
3
s0 : K

Rouths test gives K > 0 for stability. Solving


the crossings are s = 0.

(s) with K = 0,

(c)
L(s)
- Asymptotes: 2

(s + 5)(s + 7)
(s + 1)(s + 3)

2=0

- Breakin/Breakaway:
dL(s)
= 0 =) 8s3 + 64s + 104 = 0
ds
Therefore the breakin/breakaway points are at s =

2:27; 5:73.

(d)
L(s)

- Asymptotes: 3

(1 + 3s) (s + 3 + 4j) (s + 3 4j)


s(s + 1 + 2j)(s + 1 2j)
3=0

- Angle of departure:
- Angle of arrival:

= 108:4 at s =
23:4 at s =

1 + 2j

3 + 4j

2015 Pearson Education, Inc., Upper Saddle River, NJ. All rights reserved. This publication is protected by Copyright and written permission should be obtained
from the publisher prior to any prohibited reproduction, storage in a retrieval system, or transmission in any form or by any means, electronic, mechanical, photocopying,
recording, or likewise. For information regarding permission(s), write to: Rights and Permissions Department, Pearson Education, Inc., Upper Saddle River, NJ 07458.

5034

CHAPTER 5. THE ROOT-LOCUS DESIGN METHOD


Root Locus

Root Locus
5

plot a

plot b

Imag Axis

Imag Axis

-5
-5
-10

-5
Real Axis

-3

-2.5

-2

Root Locus

-1.5
-1
Real Axis

-0.5

-0.5

Root Locus
4

1.5

plot c

plot d
2
Imag Axis

Imag Axis

1
0.5
0
-0.5

-2

-1
-1.5

-4
-6

-4
Real Axis

-2

-3

-2.5

-2

-1.5
-1
Real Axis

Solution for Problem 5.20

2015 Pearson Education, Inc., Upper Saddle River, NJ. All rights reserved. This publication is protected by Copyright and written permission should be obtained
from the publisher prior to any prohibited reproduction, storage in a retrieval system, or transmission in any form or by any means, electronic, mechanical, photocopying,
recording, or likewise. For information regarding permission(s), write to: Rights and Permissions Department, Pearson Education, Inc., Upper Saddle River, NJ 07458.

5035
21. Consider the system in Fig. 5.52.

Figure 5.52: Feedback system for Problem 5.21

(a) Using Rouths stability criterion, determine all values of K for which
the system is stable.
(b) Use Matlab to nd the root locus versus K. Find the values for K
at imaginary-axis crossings.
Solution:
(a)
(s)

= s4 + 5s3 + 9s2 + (5 + K)s + 3K


s4 :
1
9
3K
s3 :
5
5+K
s2 :
8 K
3K
5
2
K
40K+200
s:
40 K
s0 :
3K

For the system to be stable, 0 < K < 4:49.


(b) The imaginary axis crossings are at s = 1:38j when K = 4:49.
Root locus is shown below.
Root Locus

Im ag Axis

-2

-4

-6

-7

-6

-5

-4

-3

-2

-1

Real Axis

Root locus for Problem 5.21

2015 Pearson Education, Inc., Upper Saddle River, NJ. All rights reserved. This publication is protected by Copyright and written permission should be obtained
from the publisher prior to any prohibited reproduction, storage in a retrieval system, or transmission in any form or by any means, electronic, mechanical, photocopying,
recording, or likewise. For information regarding permission(s), write to: Rights and Permissions Department, Pearson Education, Inc., Upper Saddle River, NJ 07458.

5036

CHAPTER 5. THE ROOT-LOCUS DESIGN METHOD

Problems and solutions for Section 5.4


22. Let
G(s) =

1
(s + 2)(s + 3)

and Dc (s) = K

s+a
:
s+b

Using root-locus techniques, nd values for the parameters a; b, and K of


the compensation Dc (s) that will produce closed-loop poles at s = 1 j
for the system shown in Fig. 5.53.

Figure 5.53: Unity feedback system for Problems 5.22, 5.28, and 5.33
Solution:
Since the desired poles are slower than the plant, we will use PI control.
The solution is to cancel the pole at -3 with the zero and set the gain to
K = 2: Thus, a = 3; b = 0; and K = 2:

2015 Pearson Education, Inc., Upper Saddle River, NJ. All rights reserved. This publication is protected by Copyright and written permission should be obtained
from the publisher prior to any prohibited reproduction, storage in a retrieval system, or transmission in any form or by any means, electronic, mechanical, photocopying,
recording, or likewise. For information regarding permission(s), write to: Rights and Permissions Department, Pearson Education, Inc., Upper Saddle River, NJ 07458.

5037
23. Suppose that in Fig. 5.53,
G(s) =

1
s(s2 + 2s + 5)

and Dc (s) =

K
:
s+2

Without using Matlab, sketch the root locus with respect to K of the
characteristic equation for the closed-loop system, paying particular attention to points that generate multiple roots. Find the value of K at
that point, state what the location of the multiple roots is, and how many
multiple roots there are.

Solution:
The root locus for the system is attached at the end.
(s) =
- Asymptotes: 4

1
s(s + 2)(s2 + 2s + 5)

0=4

- Center of asymptotes:

- Angle of asymptotoes:

45 ; 135

- Angle of departure:

90 at s =

1 + 2j

- Imaginary-axis crossings:
(s)

= s4 + 4s3 + 9s2 + 10s + K


s4 :
1
9 K
s3 :
4
10
s2 :
6:5
K
s : 10 8K
13
s0 :
K

Rouths test gives 0 < K < 16:25 for stability. Solving


K = 16:25, the crossings are s = 1:58j.

(s) with

- Location of multiple roots:


If a polynomial has repeated roots, its derivative is equal to zero at
the multiple roots. Therefore
d (s)
= 4s3 + 12s2 + 18s + 10 = 0
ds
Thus the repeated roots are at s = 1; 1 1:225j. Plugging the
roots into the characteristic equation, the corresponding value of K
is K = 4; 16:25, respectively.

2015 Pearson Education, Inc., Upper Saddle River, NJ. All rights reserved. This publication is protected by Copyright and written permission should be obtained
from the publisher prior to any prohibited reproduction, storage in a retrieval system, or transmission in any form or by any means, electronic, mechanical, photocopying,
recording, or likewise. For information regarding permission(s), write to: Rights and Permissions Department, Pearson Education, Inc., Upper Saddle River, NJ 07458.

5038

CHAPTER 5. THE ROOT-LOCUS DESIGN METHOD


Root Locus
4

I magi nary Axi s

-1

-2

-3

-4
-4

-3

-2

-1
Real Axis

Root locus for Problem 5.23

2015 Pearson Education, Inc., Upper Saddle River, NJ. All rights reserved. This publication is protected by Copyright and written permission should be obtained
from the publisher prior to any prohibited reproduction, storage in a retrieval system, or transmission in any form or by any means, electronic, mechanical, photocopying,
recording, or likewise. For information regarding permission(s), write to: Rights and Permissions Department, Pearson Education, Inc., Upper Saddle River, NJ 07458.

5039
24. Suppose the unity feedback system of Fig. 5.53 has an open-loop plant
s+z
to be
given by G(s) = 1=s2 . Design a lead compensation Dc (s) = K s+p
added in series with the plant so that the dominant poles of the closed-loop
system are located at s = 2 2j.
Solution:
Setting the pole of the lead to be at p = 20; the zero is at z =
with a gain of K = 72: The locus is plotted below.

1:78

Root Loc us
3

0.92

0.84

0.74

0.6

0.42

0.22

2
0.965

Imag Axis

0.99

7
0

-1

-6

-5

0.99

0.965
-2

-3
-7

0.84

0.92

0.74
-4

-3

0.6
-2

0.42

0.22
-1

Real Axis

Root locus for Problem 5.24

2015 Pearson Education, Inc., Upper Saddle River, NJ. All rights reserved. This publication is protected by Copyright and written permission should be obtained
from the publisher prior to any prohibited reproduction, storage in a retrieval system, or transmission in any form or by any means, electronic, mechanical, photocopying,
recording, or likewise. For information regarding permission(s), write to: Rights and Permissions Department, Pearson Education, Inc., Upper Saddle River, NJ 07458.

5040

CHAPTER 5. THE ROOT-LOCUS DESIGN METHOD

25. Assume that the unity feedback system of Fig. 5.53 has the open-loop
plant
1
G(s) =
:
s(s + 3)(s + 6)
Design a lag compensation to meet the following specications:
The step response settling time is to be less than 5 sec.
The step response overshoot is to be less than 17%.
The steady-state error to a unit ramp input must not exceed 10%.
Solution:
The overshoot specication requires that damping be 0:5 and the
settling time requires that ! n > 1:8: From the root locus plotted
below, these can be met at K = 28 where the ! n = 2: With this
gain, the Kv = 28=18 = 1:56: To get a Kv = 10; we need a lag gain
of about 6:5: Selecting the lag zero to be at 0:1 requires the pole
to be at 0:1=6:5 = 0:015: To meet the overshoot specications, it is
necessary to select a smaller K and set p = 0:01: Other choices are
of course possible. The step response of this design is plotted below.

0.84

0.74

RootLocus
0.6

1.4
0.42 0.22

0.92

1.2

Imag Axis

0.965
2
10
2

-2

0.99

0.8
10

0.6
0.99

0.4
0.965

-4

0.2
0.92

-6

0.84
-10

0.74

0.6 0.42 0.22


-5
RealAxis

00

10

15

20

Root locus and step response for Problem 5.25

2015 Pearson Education, Inc., Upper Saddle River, NJ. All rights reserved. This publication is protected by Copyright and written permission should be obtained
from the publisher prior to any prohibited reproduction, storage in a retrieval system, or transmission in any form or by any means, electronic, mechanical, photocopying,
recording, or likewise. For information regarding permission(s), write to: Rights and Permissions Department, Pearson Education, Inc., Upper Saddle River, NJ 07458.

25

5041
26. A numerically controlled machine tool positioning servomechanism has a
normalized and scaled transfer function given by
G(s) =

1
:
s(s + 1)

Performance specications of the system in the unity feedback conguration of Fig.


p 5.53 are satised if the closed-loop poles are located at
s = 1 j 3.
(a) Show that this specication cannot be achieved by choosing proportional control alone, Dc (s) = kp .
s+z
that will meet the speci(b) Design a lead compensator Dc (s) = K s+p
cation.

Solution:
(a) With proportional control, the complex poles have real part at s =
0:5.
(b) To design a lead, we select the pole to be at p =
zero and gain to be z = 3; K = 12.

10 and nd the

2015 Pearson Education, Inc., Upper Saddle River, NJ. All rights reserved. This publication is protected by Copyright and written permission should be obtained
from the publisher prior to any prohibited reproduction, storage in a retrieval system, or transmission in any form or by any means, electronic, mechanical, photocopying,
recording, or likewise. For information regarding permission(s), write to: Rights and Permissions Department, Pearson Education, Inc., Upper Saddle River, NJ 07458.

5042

CHAPTER 5. THE ROOT-LOCUS DESIGN METHOD

27. A servomechanism position control has the plant transfer function


G(s) =

10
:
s(s + 1)(s + 10)

You are to design a series compensation transfer function Dc (s) in the


unity feedback conguration to meet the following closed-loop specications:
The response to a reference step input is to have no more than 16%
overshoot.
The response to a reference step input is to have a rise time of no
more than 0.4 sec.
The steady-state error to a unit ramp at the reference input must be
less than 0.05.
(a) Design a lead compensation that will cause the system to meet the
dynamic response specications, ignoring the error requirement.
(b) What is the velocity constant Kv for your design? Does it meet the
error specication?
(c) Design a lag compensation to be used in series with the lead you
have designed to cause the system to meet the steady-state error
specication.
(d) Give the Matlab plot of the root locus of your nal design.
(e) Give the Matlab response of your nal design to a reference step.
Solution:
(a) Setting the lead pole at p = 60 and the zero at z = 1; the dynamic
specications are met with a gain of 245. With the lead compensator,
the overshoot is reduced to 3:64% and the rise time is 0:35 sec.
(b)
Kv = lim sGDc = lim s
s!0

s!0

245(s + 1)
10
= 4:083
s(s + 1)(s + 10) (s + 6)

(c) To meet the steady-state requirement, we need a new Kv = 20; which


is an increase of a factor of 5. If we set the lag zero at z = 0:4; the
pole needs to be at p = 0:08:
(d) The root locus is plotted below.
(e) The step response is plotted below.

2015 Pearson Education, Inc., Upper Saddle River, NJ. All rights reserved. This publication is protected by Copyright and written permission should be obtained
from the publisher prior to any prohibited reproduction, storage in a retrieval system, or transmission in any form or by any means, electronic, mechanical, photocopying,
recording, or likewise. For information regarding permission(s), write to: Rights and Permissions Department, Pearson Education, Inc., Upper Saddle River, NJ 07458.

5043

Lead root locus

4
3

Lead Step response


1.5

0.92

0.84

0.74

0.6

0.42 0.22

0.965
1

0.99
1
10
0

Amplitude

Imaginary Axis

-1
0.99

0.5

-2
-3
-4

0.965

0.84

0.92

-10

0.74

0.6

0.42 0.22
0

-8

-6

-4
Real Axis

-2

0.5

Lead-lag root locus

1.5

Lead-lag step response


1.5

4 0.92
3

1
Time (sec)

0.84

0.74

0.6

0.42 0.22

0.965
1

0.99
1
10
0

Amplitude

Imaginary Axis

-1
0.99

0.5

-2
-3
-4
-10

0.965

0.84

0.92

0.74

0.6

0.42 0.22
0

-8

-6

-4
Real Axis

-2

3
4
Time (sec)

Solution to Problem 5.27

2015 Pearson Education, Inc., Upper Saddle River, NJ. All rights reserved. This publication is protected by Copyright and written permission should be obtained
from the publisher prior to any prohibited reproduction, storage in a retrieval system, or transmission in any form or by any means, electronic, mechanical, photocopying,
recording, or likewise. For information regarding permission(s), write to: Rights and Permissions Department, Pearson Education, Inc., Upper Saddle River, NJ 07458.

5044

CHAPTER 5. THE ROOT-LOCUS DESIGN METHOD

28. Assume the closed-loop system of Fig. 5.53 has a feed forward transfer
function G(s) given by
1
:
G(s) =
s(s + 2)
Design a lag compensation so that the dominant poles of the closed-loop
system are located at s = 1 j and the steady-state error to a unit ramp
input is less than 0.2.
Solution:
The poles can be put in the desired location with proportional control
alone, with a gain of kp = 2 resulting in a Kv = 1: To get a Kv = 5; we add
s + 0:1
:
a compensation with zero at 0:1 and a pole at 0:02: Dc (s) = 2
s + 0:02

2015 Pearson Education, Inc., Upper Saddle River, NJ. All rights reserved. This publication is protected by Copyright and written permission should be obtained
from the publisher prior to any prohibited reproduction, storage in a retrieval system, or transmission in any form or by any means, electronic, mechanical, photocopying,
recording, or likewise. For information regarding permission(s), write to: Rights and Permissions Department, Pearson Education, Inc., Upper Saddle River, NJ 07458.

5045
29. An elementary magnetic suspension scheme is depicted in Fig. 5.54. For
small motions near the reference position, the voltage e on the photo
detector is related to the ball displacement x(in meters) by e = 100x.
The upward force (in newtons) on the ball caused by the current i (in
amperes) may be approximated by f = 0:5i + 20x. The mass of the ball
is 20 g, and the gravitational force is 9.8 N/kg. The power amplier is a
voltage-to-current device with an output (in amperes) of i = u + V0 .

Figure 5.54: Elementary magnetic suspension

(a) Write the equations of motion for this setup.


(b) Give the value of the bias V0 that results in the ball being in equilibrium at x = 0.
(c) What is the transfer function from u to e?
(d) Suppose the control input u is given by u = Ke. Sketch the root
locus of the closed-loop system as a function of K.
U
(e) Assume that a lead compensation is available in the form E
=
s+z
Dc (s) = K s+p : Give values of K; z; and p that yields improved
performance over the one proposed in part (d).
Solution:
(a) The equations of motion can be written as
X
m
x =
f orces
=

0:5i + 20x

mg = 0:5(u + Vo ) + 20x

mg

Substituting numbers, we have


0:02
x = 0:5(u + Vo ) + 20x

0:196:

2015 Pearson Education, Inc., Upper Saddle River, NJ. All rights reserved. This publication is protected by Copyright and written permission should be obtained
from the publisher prior to any prohibited reproduction, storage in a retrieval system, or transmission in any form or by any means, electronic, mechanical, photocopying,
recording, or likewise. For information regarding permission(s), write to: Rights and Permissions Department, Pearson Education, Inc., Upper Saddle River, NJ 07458.

5046

CHAPTER 5. THE ROOT-LOCUS DESIGN METHOD


(b) In equilibrium at x = 0, x
= 0 and u = 0. Therefore to have the bias
cancel gravity, 0:5Vo 0:196 = 0 or Vo = 0:392:
(c) Taking Laplace transforms of the equation and substituting e = 100x;

E
2500
= 2
U
s
1000

(d) The locus starts at the two poles symmetric to the imaginary axis,
meet at the origin and cover the imaginary axis. The locus is plotted
below.

Root Locus
40

30

Imaginary Axis

20

10

-10

-20

-30

-40
-40

-30

-20

-10

0
Real Axis

10

20

30

40

Root loci for Problem 5.29d

(e) Since the system with a proportional gain is on the stability boundary, any p
lead will improve its performance. For example, we can
pick z = 1000 to cancel one of the open-loop plant poles, and pick
p = 150 to pull the locus into the left-hand plane. K can be selected
to give a desired amount of damping, say 0:7. K = 4:75 gives a
damping of 0:7. See the plot below.

2015 Pearson Education, Inc., Upper Saddle River, NJ. All rights reserved. This publication is protected by Copyright and written permission should be obtained
from the publisher prior to any prohibited reproduction, storage in a retrieval system, or transmission in any form or by any means, electronic, mechanical, photocopying,
recording, or likewise. For information regarding permission(s), write to: Rights and Permissions Department, Pearson Education, Inc., Upper Saddle River, NJ 07458.

5047

Root Locus
50
40
30

Imaginary Axis

20
10
0
-10
-20
-30
-40
-50
-160

-140

-120

-100

-80

-60
Real Axis

-40

-20

20

40

Root loci for Problem 5.29e

2015 Pearson Education, Inc., Upper Saddle River, NJ. All rights reserved. This publication is protected by Copyright and written permission should be obtained
from the publisher prior to any prohibited reproduction, storage in a retrieval system, or transmission in any form or by any means, electronic, mechanical, photocopying,
recording, or likewise. For information regarding permission(s), write to: Rights and Permissions Department, Pearson Education, Inc., Upper Saddle River, NJ 07458.

5048

CHAPTER 5. THE ROOT-LOCUS DESIGN METHOD

30. A certain plant with the non minimum phase transfer function
G(s) =

4 2s
;
s2 + s + 9

is in a unity positive feedback system with the controller transfer function


Dc (s):
(a) Use Matlab to determine a (negative) value for Dc (s) = K so that
the closed-loop system with negative feedback has a damping ratio
= 0:707.
(b) Use Matlab to plot the systems response to a reference step.
Solution:
(a) With all the negatives, the problem statement might be confusing.
With the G(s) as given, Matlab needs to plot the negative locus,
which is the regular positive locus for G: The locus is plotted below.
The value of gain for closed loop roots at damping of 0:7 is K = 1:04
(b) The nal value of the step response plotted below is 0:887. To get
a positive output we would use a positive gain in positive feedback.
RootLocus
0.84

4 0.92
3

0.74

0.6

StepResponse

0.4

0.42 0.22

0.2
0.965

0
Amplitude

Imag Axis

0.99
1
100

-0.2

-0.4

-1
0.99
-0.6

-2
-3
-4
-10

0.965
-0.8
0.92

0.84
-8

0.74
-6

0.6

-4
RealAxis

0.42 0.22
-2

-1

2
Time(sec)

Solutions for Problem 5.30

2015 Pearson Education, Inc., Upper Saddle River, NJ. All rights reserved. This publication is protected by Copyright and written permission should be obtained
from the publisher prior to any prohibited reproduction, storage in a retrieval system, or transmission in any form or by any means, electronic, mechanical, photocopying,
recording, or likewise. For information regarding permission(s), write to: Rights and Permissions Department, Pearson Education, Inc., Upper Saddle River, NJ 07458.

5049
31. Consider the rocket-positioning system shown in Fig. 5.55.

Figure 5.55: Block diagram for rocket-positioning control system

(a) Show that if the sensor that measures x has a unity transfer function,
the lead compensator
s+2
H(s) = K
s+4
stabilizes the system.
(b) Assume that the sensor transfer function is modeled by a single pole
with a 0:1 sec time constant and unity DC gain. Using the root-locus
procedure, nd a value for the gain K that will provide the maximum
damping ratio.
Solution:
(a) The root locus is plotted below and lies entirely in the left-half plane.
So the system is stable for all K.
(b) At maximum damping, the gain is K = 6:25 but the damping of the
complex poles is only 0:073: A practical design would require much
more lead.
RootLocus
0.19
0.13

0.26
10

0.09 0.06 0.03 12


10

0.4

0.65

0.4
0.26

0.19
-3

0.13
-2
RealAxis

1.5

0.09 0.06 0.03 12


-1
0

0.5

0.5

0.97
-1

1
0.88

1.5

0.76

2.5

-2

10
-4

0.88

6
-10

3
2.5

0.97

2
-5

RootLocus
0.36 0.24 0.12

1
Imag Axis

Imag Axis

4
2

0.48

5 0.65

0.62
0.76

-3-3

2
0.62

0.48
-2

0.36 0.24 0.12


-1
RealAxis

30

Loci for Problem 5.31

2015 Pearson Education, Inc., Upper Saddle River, NJ. All rights reserved. This publication is protected by Copyright and written permission should be obtained
from the publisher prior to any prohibited reproduction, storage in a retrieval system, or transmission in any form or by any means, electronic, mechanical, photocopying,
recording, or likewise. For information regarding permission(s), write to: Rights and Permissions Department, Pearson Education, Inc., Upper Saddle River, NJ 07458.

5050

CHAPTER 5. THE ROOT-LOCUS DESIGN METHOD

32. For the system in Fig. 5.56:

Figure 5.56: Control system for Problem 5.32

(a) Find the locus of closed-loop roots with respect to K.


(b) Find the maximum value of K for which the system is stable. Assume
K = 2 for the remaining parts of this problem.
(c) What is the steady-state error (e = r y) for a step change in r?
(d) What is the steady-state error in y for a constant disturbance w1 ?
(e) What is the steady-state error in y for a constant disturbance w2 ?
(f) If you wished to have more damping, what changes would you make
to the system?
Solution:
(a) For the locus, L(s) =

100(s + 1)
: The locus is plotted below.
s2 (s2 + 12s + 40)
Root Locus

0.92

0.84

0.74

0.6

0.42

0.22

4
0.965

Im ag Axis

0.99

14
0

-2

-4

-6
-14

12

10

-12

-10

0.99

0.965

0.84

0.92

0.74
-8

-6

0.6
-4

0.42

0.22
-2

Real Axis

Locus for Problem 5.32

2015 Pearson Education, Inc., Upper Saddle River, NJ. All rights reserved. This publication is protected by Copyright and written permission should be obtained
from the publisher prior to any prohibited reproduction, storage in a retrieval system, or transmission in any form or by any means, electronic, mechanical, photocopying,
recording, or likewise. For information regarding permission(s), write to: Rights and Permissions Department, Pearson Education, Inc., Upper Saddle River, NJ 07458.

5051
(b) The maximum value of K for stability is K = 3:36:
(c) The transfer function from R to Y is
Y
200
= 2 2
:
R
s (s + 12s + 40) + 200(s + 1)
Therefore the steady-state error for a step change in r is
estep (1)

Y 1
R s
2 2
s (s + 12s + 40) + 200s
=0
= lim 2 2
s!0 s (s + 12s + 40) + 200(s + 1)
=

lim s 1

s!0

(d) The transfer function from W1 to Y is:


Y
100s2
= 2 2
W1
s (s + 12s + 40) + 200(s + 1)
Therefore the steady-state error for a constant disturbance w1 is
estep (1) = lim s
s!0

Y
W1

1
=0
s

(e) The transfer function from W2 to Y is:


Y
100
= 2 2
W2
s (s + 12s + 40) + 200(s + 1)
Therefore the steady-state error for a constant disturbance w1 is
estep (1) = lim s
s!0

Y
W2

1
= 0:5
s

(f) To get more damping in the closed-loop response, the controller needs
to have a lead compensation.

2015 Pearson Education, Inc., Upper Saddle River, NJ. All rights reserved. This publication is protected by Copyright and written permission should be obtained
from the publisher prior to any prohibited reproduction, storage in a retrieval system, or transmission in any form or by any means, electronic, mechanical, photocopying,
recording, or likewise. For information regarding permission(s), write to: Rights and Permissions Department, Pearson Education, Inc., Upper Saddle River, NJ 07458.

5052

CHAPTER 5. THE ROOT-LOCUS DESIGN METHOD

33. Consider the plant transfer function


G(s) =

bs + k
s2 [mM s2 + (M + m)bs + (M + m)k]

to be put in the unity feedback loop of Fig. 5.53. This is the transfer
function relating the input force u(t) and the position y(t) of mass M in
the non-collocated sensor and actuator problem. In this problem, we will
use root-locus techniques to design a controller Dc (s) so that the closedloop step response has a rise time of less than 0.1 sec and an overshoot of
less than 10%. You may use Matlab for any of the following questions:
(a) Approximate G(s) by assuming that m = 0, and let M = 1, k = 1,
b = 0:1, and Dc (s) = K. Can K be chosen to satisfy the performance
specications? Why or why not?
(b) Repeat part (a) assuming Dc (s) = K(s + z), and show that K and z
can be chosen to meet the specications.
(c) Repeat part (b) but with a practical controller given by the transfer
function
p(s + z)
;
Dc (s) = K
s+p
and pick p so that the values for K and z computed in part (b) remain
more or less valid.
(d) Now suppose that the small mass m is not negligible, but is given by
m = M=10. Check to see if the controller you designed in part (c)
still meets the given specications. If not, adjust the controller parameters so that the specications are met.
Solution:
(a) The approximate plant transfer function is G(s) = s12 . The locus in
this case is the imaginary axis and cannot meet the specs for any K:
(b) The specs require that > 0:6; ! n > 18: Select z = 15 for a start.
The locus will be a circle with radius 15: Because of the zero, the
overshoot will be increased and Figure 3.29 indicates that wed better
make the damping greater than 0.7. As a matter of fact, experimentation shows that we can lower the overshoot of less than 10% only
by setting the zero at a low value and putting the poles on the real
axis. The plot shows the result when Dc = 25(s + 4): The resulting
overshoot is 9:9% and the rise time is 0:06 sec.
(c) In this case, we pick p = 150; z = 4; and K = 30. Then the resulting
overshoot is 9:8% and the rise time is 0:05 sec.
(d) With the resonance present, the only chance we have is to introduce
a notch as well as a lead. The compensation resulting in the plots
s + 4 s2 =9:25 + s=9:25 + 1
: The overshoot
shown is Dc (s) = 12
(:01s + 1) s2 =3600 + s=30 + 1
is 7% and the rise time is 0:04 sec :

2015 Pearson Education, Inc., Upper Saddle River, NJ. All rights reserved. This publication is protected by Copyright and written permission should be obtained
from the publisher prior to any prohibited reproduction, storage in a retrieval system, or transmission in any form or by any means, electronic, mechanical, photocopying,
recording, or likewise. For information regarding permission(s), write to: Rights and Permissions Department, Pearson Education, Inc., Upper Saddle River, NJ 07458.

5053

Root Locus

1.5

Imaginary Axis

20

10
0

0.5
-10
-20
-60

-50

-40

-30
Real Axis

-20

-10

Root Locus

0.2

0.4

0.6

0.8

0.2

0.4

0.6

0.8

1.5

Imaginary Axis

20

10
0

0.5
-10
-20
-60

-50

-40

-30
Real Axis

-20

-10

Root Locus

1.5

Imaginary Axis

0.5
-2

-4
-10

-8

-6

-4
Real Axis

-2

0.5

1.5

2.5

3.5

Root loci and step responses for Problem 5.33

2015 Pearson Education, Inc., Upper Saddle River, NJ. All rights reserved. This publication is protected by Copyright and written permission should be obtained
from the publisher prior to any prohibited reproduction, storage in a retrieval system, or transmission in any form or by any means, electronic, mechanical, photocopying,
recording, or likewise. For information regarding permission(s), write to: Rights and Permissions Department, Pearson Education, Inc., Upper Saddle River, NJ 07458.

5054

CHAPTER 5. THE ROOT-LOCUS DESIGN METHOD

34. Consider the Type 1 system drawn in Fig. 5.57. We would like to design the compensation Dc (s) to meet the following requirements: (1) The
steady-state value of y due to a constant unit disturbance w should be less
than 54 , and (2) the damping ratio = 0:7. Using root-locus techniques,

Figure 5.57: Control system for Problem 5.34

(a) Show that proportional control alone is not adequate.


(b) Show that proportional-derivative control will work.
(c) Find values of the gains kp and kD for Dc (s) = kp + kD s that meet
the design specications with at least a 10% margin.
Solution:
(a) To meet the error requirement, we need
ystep (1) = lim s
s!0

1
Y 1
= lim
W s s!0 s2 + s + K

0:8:

Thus K must be at least K 1:25. With this K, the damping ratio


will be
1
= p
)
0:45
2 K
So we cant meet both requirements with proportional control.
(b) With PD control,
ystep (1) = lim s
s!0

Y 1
1
= lim
W s s!0 s2 + (1 + kD )s + kp

So the error requirement can be met by setting kp


damping ratio requirement can be written as
=
By choosing kD

p
1:4 kp

1 + kD
p
2 kp

0:8:
1:25. The

0:7

1, we can satisfy both specications.

2015 Pearson Education, Inc., Upper Saddle River, NJ. All rights reserved. This publication is protected by Copyright and written permission should be obtained
from the publisher prior to any prohibited reproduction, storage in a retrieval system, or transmission in any form or by any means, electronic, mechanical, photocopying,
recording, or likewise. For information regarding permission(s), write to: Rights and Permissions Department, Pearson Education, Inc., Upper Saddle River, NJ 07458.

5055
(c) Setting kp = 1:4 and kD = 0:85, we get ystep (1) = 0:714 and =
0:782. The root loci and disturbance step response are plotted below.

R o ot L oc u s

Ste p r e s p o n s e fo r p r o b le m 5 .3 4

1 .5

0 .8
0 .9 6

0 .9 2

0 .8 6

0 .7 6

0 .5 8

0 .3 5

0 .7
1
0 .9 8 4
0 .6

0 .5

0 .9 9 6

Amplitude

Imaginary Axis

0 .5

5
0

0 .4

0 .3
0 .9 9 6
- 0 .5

0 .2
0 .9 8 4
-1
0 .1

0 .9 2

0 .9 6
- 1 .5
-5

0 .8 6

0 .7 6

0 .5 8

0 .3 5
0

-4

-3

-2

-1

R e a l Ax is

Time ( s e c )

Solution for Problem 5.34

2015 Pearson Education, Inc., Upper Saddle River, NJ. All rights reserved. This publication is protected by Copyright and written permission should be obtained
from the publisher prior to any prohibited reproduction, storage in a retrieval system, or transmission in any form or by any means, electronic, mechanical, photocopying,
recording, or likewise. For information regarding permission(s), write to: Rights and Permissions Department, Pearson Education, Inc., Upper Saddle River, NJ 07458.

5056

CHAPTER 5. THE ROOT-LOCUS DESIGN METHOD

35. Using a sample rate of 10 Hz, nd the Dc (z) that is the discrete equivalent
to your Dc (s) from Problem 5.34 using the trapezoid rule. Evaluate the
time reponse using Simulink, and determine whether the damping ratio
requirement is met with the digital implementation. (Note: The material
to do this problem is covered in the Appendix W4.5 at www.FPE7e.com
or in Chapter 8.) (Note: The rst printing of the 7th edition had an
error in the problem statement. It said to use the Dc (s) from Problem
5.7 rather than the correct one, Problem 5.34)
Solution:
From Problem 5.34, we have Dc (s) = 0:85s + 1:4. The discrete equivalent
2 z 1
in the Dc (s):
for Ts = 0:1 sec is given by substituting s =
0:1 z + 1
Dd (z) = 0:85

2 z 1
18:4z 15:6
+ 1:4 =
:
0:1 z + 1
z+1

To evaluate this discrete controller, we use Simulink to compare the two


implementations. The results of the step responses are shown below.

1
Continuousdesign
Discreteequivalentdesign,T

0.8

s =0.1sec

0.6

0.4

0.2

00

Time(sec)

10

12

14

Solution for Problem 5.35


Note that there is slightly greater overshoot in the digital system, which
suggests a decrease in the damping due to the digital implementation.
However, the damping ratio requirement of 0:7 is still met with the digital
control. It is interesting to note, however, that there is an oscillation of
the output at 5 Hz, which is half the sample rate. This arises because the
use of the trapezoidal equivalent places a root of the compensation at z
= -1, which has created an extraneous oscillatory root in the closed-loop
system. .This is also a result of having a pure dierentiation in the D(s),

2015 Pearson Education, Inc., Upper Saddle River, NJ. All rights reserved. This publication is protected by Copyright and written permission should be obtained
from the publisher prior to any prohibited reproduction, storage in a retrieval system, or transmission in any form or by any means, electronic, mechanical, photocopying,
recording, or likewise. For information regarding permission(s), write to: Rights and Permissions Department, Pearson Education, Inc., Upper Saddle River, NJ 07458.

5057
i.e., a zero in the numerater with no pole. Using the Matched Pole-Zero
discrete equivalent will help this situation since it does not create the
oscillatory pole in the compensator. Using Matlabs C2D function for
this discrete equivalent produces
Dd2 (z) = 9:219z

7:819:

which eliminates the oscillation at 5 hz.

2015 Pearson Education, Inc., Upper Saddle River, NJ. All rights reserved. This publication is protected by Copyright and written permission should be obtained
from the publisher prior to any prohibited reproduction, storage in a retrieval system, or transmission in any form or by any means, electronic, mechanical, photocopying,
recording, or likewise. For information regarding permission(s), write to: Rights and Permissions Department, Pearson Education, Inc., Upper Saddle River, NJ 07458.

5058

CHAPTER 5. THE ROOT-LOCUS DESIGN METHOD

Problems and solutions for Section 5.5


36. Consider the positioning servomechanism system shown in Fig. 5.58, where
ei = Ko i ; eo = Ko o ; Ko = 10V=rad;
(note: 1st printing of book had eo = Kpot o ; which is not correct)
T = motor torque = Kt ia ;
km = Kt = torque constant = 0:1 N m=A;
Ke = back emf constant = 0:1V sec
Ra = armature resistance = 10 ;
Gear ratio = 1 : 1;
JL + Jm = total inertia = 10
va = KA (ei ef ):

kg m2 ;

Figure 5.58: Positioning servomechanism

(a) What is the range of the amplier gain KA for which the system is
stable? Estimate the upper limit graphically using a root-locus plot.
(b) Choose a gain KA that gives roots at = 0:7. Where are all three
closed-loop root locations for this value of KA ?
Solution:
(a) Neglecting viscous friction and the eect of inductance, the transfer
function of the DC motor is
o

Va

(JL + Jm

Kt
Ra
)s2

Kt Ke
Ra s

10
s2 + s

2015 Pearson Education, Inc., Upper Saddle River, NJ. All rights reserved. This publication is protected by Copyright and written permission should be obtained
from the publisher prior to any prohibited reproduction, storage in a retrieval system, or transmission in any form or by any means, electronic, mechanical, photocopying,
recording, or likewise. For information regarding permission(s), write to: Rights and Permissions Department, Pearson Education, Inc., Upper Saddle River, NJ 07458.

5059
From the root locus plotted below, the upper limit of KA for stability
is 0:11.
Root Locus
3

0.92

0.84

0.74

0.6

0.42

0.22

2
0.965

Imaginary A xis

0.99

7
0

-1

-6

-5

0.99

0.965
-2

-3
-7

0.84

0.92

0.74
-4

-3
Real Axis

0.6
-2

0.42

0.22
-1

Root locus for Problem 5.36


(b) The damping is 0:7 when K = 0:046. For the value of KA , poles are
at s = 10:05; 0:475 0:482j:

2015 Pearson Education, Inc., Upper Saddle River, NJ. All rights reserved. This publication is protected by Copyright and written permission should be obtained
from the publisher prior to any prohibited reproduction, storage in a retrieval system, or transmission in any form or by any means, electronic, mechanical, photocopying,
recording, or likewise. For information regarding permission(s), write to: Rights and Permissions Department, Pearson Education, Inc., Upper Saddle River, NJ 07458.

5060

CHAPTER 5. THE ROOT-LOCUS DESIGN METHOD

37. We wish to design a velocity control for a tape-drive servomechanism. The


transfer function from current I(s) to tape velocity (s) (in millimeters
per millisecond per ampere) is
15(s2 + 0:9s + 0:8)
(s)
=
:
I(s)
(s + 1)(s2 + 1:1s + 1)
We wish to design a Type 1 feedback system so that the response to a
reference step satises
tr

4msec;

ts

15msec;

Mp

0:05

(a) Use the integral compensator kI =s to achieve Type 1 behavior, and


sketch the root-locus with respect to kI . Show on the same plot the
region of acceptable pole locations corresponding to the specications.
(b) Assume a proportional-integral compensator of the form kp (s + )=s,
and select the best possible values of kp and you can nd. Sketch
the root-locus plot of your design, giving values for kp and , and
the velocity constant Kv your design achieves. On your plot, indicate
the closed-loop poles with a dot ( ) and include the boundary of the
region of acceptable root locations.
Solution:

(a) The root locus with respect to kI and the step response with kI =
0:036 are plotted in the rst row below.
(b) Using rltool, we can choose the location of zeroto pull the locus to
the left-hand plane. This will improve the transient response. Here,
the zero was put at s = 1:7, and kp was set to 0:036. The root
locus and the step response are plotted in the second row below.
With the PI compensator, the closed-loop poles are at s = 0:7749
0:7774j; 0:5451 0:5589j, and the closed-loop zeros are at s = 1:7;
0:45 0:773j. Thus the velocity constant can be calculated from
Truxals formula.
n

X
1
=
Kv
i=1

X 1
1
+
pi i=1 zi

Kv = 0:7344

2015 Pearson Education, Inc., Upper Saddle River, NJ. All rights reserved. This publication is protected by Copyright and written permission should be obtained
from the publisher prior to any prohibited reproduction, storage in a retrieval system, or transmission in any form or by any means, electronic, mechanical, photocopying,
recording, or likewise. For information regarding permission(s), write to: Rights and Permissions Department, Pearson Education, Inc., Upper Saddle River, NJ 07458.

5061

Integ r al c ontr ol with k= 0.036

R oot loc us

1.2
1
0.8
Amplitude

Imaginary Axis

0.5

0.6
0.4

- 0.5
0.2
-1
- 1.5

0
-1

- 0.5

0.5

R eal Axis

10

10

T ime ( sec )

PI c ontr ol with k = 0.036

R oot Locus

0.8
0.6

1
0.8
Amplitude

Imaginary Axis

0.4
0.2
0

0.6

- 0.2
0.4
- 0.4
- 0.6

0.2

- 0.8
0
-2

- 1.5

-1

- 0.5

R eal Axis

T ime ( sec )

Solution for Problem 5.37

2015 Pearson Education, Inc., Upper Saddle River, NJ. All rights reserved. This publication is protected by Copyright and written permission should be obtained
from the publisher prior to any prohibited reproduction, storage in a retrieval system, or transmission in any form or by any means, electronic, mechanical, photocopying,
recording, or likewise. For information regarding permission(s), write to: Rights and Permissions Department, Pearson Education, Inc., Upper Saddle River, NJ 07458.

5062

CHAPTER 5. THE ROOT-LOCUS DESIGN METHOD

38. The normalized, scaled equations of a cart as drawn in Fig. 5.59 of mass
mc holding an inverted uniform pendulum of mass mp and length ` with
no friction are

= v
(5.88)
y +
=v
where

3mp
4(mc +mp )

is a mass ratio bounded by 0 <

< 0:75. Time is

3g(m +m )
measured in terms of = ! o t where ! 2o = `(4mcc+mpp) : The cart motion, y;
is measured in units of pendulum length as y = 3x
4` and the input is force
normalized by the system weight, v = g(mcu+mp ) : These equations can be

used to compute the transfer functions


V

1
s2

s2 1 +
Y
= 2 2
V
s (s
1)

(5.89)
(5.90)

In this problem you are to design a control for the system by rst closing
a loop around the pendulum, Eq.(5.89) and then, with this loop closed,
closing a second loop around the cart plus pendulum, Eq.(5.90). For this
problem, let the mass ratio be mc = 5mp :

Figure 5.59: Figure of cart-pendulum for Problem 5.38

(a) Draw a block diagram for the system with V input and both Y and
as outputs.
s+z
for the loop to cancel
(b) Design a lead compensation Dc (s) = K
s+p
the pole at s = 1 and place the two remaining poles at 4 j4: The
new control is U (s), where the force is V (s) = U (s) + Dc (s) (s):
Draw the root locus of the angle loop.
(c) Compute the transfer function of the new plant from U to Y with
Dc (s) in place.

2015 Pearson Education, Inc., Upper Saddle River, NJ. All rights reserved. This publication is protected by Copyright and written permission should be obtained
from the publisher prior to any prohibited reproduction, storage in a retrieval system, or transmission in any form or by any means, electronic, mechanical, photocopying,
recording, or likewise. For information regarding permission(s), write to: Rights and Permissions Department, Pearson Education, Inc., Upper Saddle River, NJ 07458.

5063
(d) Design a controller Dc (s) for the cart position with the pendulum
loop closed. Draw the root locus with respect to the gain of Dc (s)
(e) Use Matlab to plot the control, cart position, and pendulum position
for a unit step change in cart position.

Solution:

(a)

(b) To cancel the pole at s = 1, we set z = 1. Then the closed loop


transfer function for the loop becomes

(s

K
= 2
1)(s + p) + K
s + (p

K
1)s + K

Setting p = 9 and K = 41, the remaining poles can be placed at


s+1
The root locus is shown
s = 4 4j. Therefore Dc (s) = 41
s+9
below.
(c) Since mc = 5mp , = 0:125. Therefore the transfer function from U
to Y with Dc (s) is
Y
41
s2 0:875
= 2
U
s + 8s + 32
s2

(d) Dc = kc

s2 + 0:2s + 0:01
: The root locus is shown below.
s2 + 2s + 1

(e) The step responses are shown below. The pendulum position control
is rather fast for this problem. A more reasonable alternative choice
would be to place the pendulum roots at s = 0:5 j0:5:

2015 Pearson Education, Inc., Upper Saddle River, NJ. All rights reserved. This publication is protected by Copyright and written permission should be obtained
from the publisher prior to any prohibited reproduction, storage in a retrieval system, or transmission in any form or by any means, electronic, mechanical, photocopying,
recording, or likewise. For information regarding permission(s), write to: Rights and Permissions Department, Pearson Education, Inc., Upper Saddle River, NJ 07458.

5064

CHAPTER 5. THE ROOT-LOCUS DESIGN METHOD


Inner pendulum loop

Outer cart loop

Final design

-2

Amplitude

Imag Axis

Imag Axis

1.5
4

-2

-4

0.5

-4
-0.5

-10

-5

-8

-6

-4

-2

Real Axis

Real Axis

Alternative design

Alternative design

50

100

150

T ime (sec)

1.5
1.2

Amplitude

Imag Axis

1
0.5
0
-0.5

0.8
0.6
0.4
0.2

-1
-1.5
-2

0
-0.2
-1

0
Real Axis

20

40

60

80

T ime (sec)

Root loci and step responses for Problem 5.38

2015 Pearson Education, Inc., Upper Saddle River, NJ. All rights reserved. This publication is protected by Copyright and written permission should be obtained
from the publisher prior to any prohibited reproduction, storage in a retrieval system, or transmission in any form or by any means, electronic, mechanical, photocopying,
recording, or likewise. For information regarding permission(s), write to: Rights and Permissions Department, Pearson Education, Inc., Upper Saddle River, NJ 07458.

5065
39. Consider the 270-ft U.S. Coast Guard cutter Tampa (902) shown in Fig. 5.60.
Parameter identication based on sea-trials data (Trankle, 1987) was used
to estimate the hydrodynamic coe cients in the equations of motion. The
result is that the response of the heading angle of the ship to rudder
angle and wind changes w can be described by the second-order transfer
functions

0:0184(s + 0:0068)
(s)
=
;
(s)
s(s + 0:2647)(s + 0:0063)
(s)
0:0000064
Gw (s) =
=
;
w(s)
s(s + 0:2647)(s + 0:0063)
G (s) =

where
= heading angle, rad
r = reference heading angle; rad:
r = yaw rate; rad=sec;
= rudder angle; rad;
w = wind speed; m=sec:

Figure 5.60: USCG cutter Tampa (902)

2015 Pearson Education, Inc., Upper Saddle River, NJ. All rights reserved. This publication is protected by Copyright and written permission should be obtained
from the publisher prior to any prohibited reproduction, storage in a retrieval system, or transmission in any form or by any means, electronic, mechanical, photocopying,
recording, or likewise. For information regarding permission(s), write to: Rights and Permissions Department, Pearson Education, Inc., Upper Saddle River, NJ 07458.

5066

CHAPTER 5. THE ROOT-LOCUS DESIGN METHOD


(a) Determine the open-loop settling time of r for a step change in .
(b) In order to regulate the heading angle , design a compensator that
uses and the measurement provided by a yaw-rate gyroscope (that
is, by _ = r). The settling time of to a step change in r is specied
to be less than 50 sec, and, for a 5 change in heading the maximum
allowable rudder angle deection is specied to be less than 10 .
(c) Check the response of the closed-loop system you designed in part (b)
to a wind gust disturbance of 10 m= sec (Model the disturbance as
a step input.) If the steady-state value of the heading due to this
wind gust is more than 0:5 , modify your design so that it meets this
specication as well.
Solution:
(a) To determine the open-loop settling time to 1% of the nal value, we
can use the stepinfo command in Matlab. The value from Matlab is
ts = 314:64 sec.
(b) The rate feedback from a yaw-rate gyroscope is giving us a derivative
control for free. Thus the block diagram of the system will look like
shown below.

Block diagram for Problem 5.39b


Lets set KD = 1 and see what we can do with a simple proportional
controller, i.e. Dc (s) = Kp . (Since the plant has a negative sign, the
controller gain will also be negative.) With the proportional control,
the maximum deection of the rudder is almost surely at the initial
instant, when it is (0) = Kp r (0): Thus to keep below 10 for a
step of 5 ; we need jKp j < 2. And for a settling time less than 50 sec,
we need > 4:6=50 = 0:092 from the design relations for the standard
second order system. Using root-locus technique, Kp = 1:5 was
picked. Checking the step response with this proportional control
using Matlab, we nd ts = 28:2 sec, which meets the settling time
requirement. And the rudder angle deection for a 5 input is less
than 7:5 . Therefore Dc (s) = 1:5 is adequate for the problem.

2015 Pearson Education, Inc., Upper Saddle River, NJ. All rights reserved. This publication is protected by Copyright and written permission should be obtained
from the publisher prior to any prohibited reproduction, storage in a retrieval system, or transmission in any form or by any means, electronic, mechanical, photocopying,
recording, or likewise. For information regarding permission(s), write to: Rights and Permissions Department, Pearson Education, Inc., Upper Saddle River, NJ 07458.

5067

Root Locus
1

Imaginary Axis

0.5

-0.5

-1
-4

-3

-2

-1
Real Axis

Heading angle to a step input

Rudder angle to 5

1.4

input

1.2
0

1
0.8
0.6

-5

0.4
0.2
0

10

20
Time (sec)

30

40

-10

10

20
Time (sec)

30

40

Root locus and Step response for Problem 5.39b

(c) With the compensator from part (b), the closed-loop transfer function from w to is

(s)
Gw
=
w(s)
1 + Kp G (1 + KD s)
=

0:0000064
s(s + 0:2647)(s + 0:0063) + 0:0278(s + 0:0068)(1 + s)

Using the Final Value Theorem, the steady-state value of the heading
angle due to a disturbance of 10m= sec is

(1) = lim s
s!0

(s) 10
= 0:3386 rad = 19:4 > 0:5
w(s) s

So we need to modify our design. To reject the disturbance completely, lets add an integral term to the controller, Dc (s) = Kp +
KI =s. Using rltool, we nd that when Dc (s) = 1:87 0:11=s, all
specications are met as shown below.

2015 Pearson Education, Inc., Upper Saddle River, NJ. All rights reserved. This publication is protected by Copyright and written permission should be obtained
from the publisher prior to any prohibited reproduction, storage in a retrieval system, or transmission in any form or by any means, electronic, mechanical, photocopying,
recording, or likewise. For information regarding permission(s), write to: Rights and Permissions Department, Pearson Education, Inc., Upper Saddle River, NJ 07458.

5068

CHAPTER 5. THE ROOT-LOCUS DESIGN METHOD

Root Locus

Heading angle to 10 m/s disturbance


0.2

0.8
0.15

0.4
0.2

P control

Imaginary Axis

0.6

0.1

PI control

-0.2
-0.4

0.05

-0.6
-0.8
-2

-1.5

-1
Real Axis

-0.5

Heading angle to a step input

200

400

600
800
Time (sec)

Rudder angle to 5

1.4

1000

1200

1400

input

1.2
0

1
0.8
0.6

-5

0.4
0.2
0

20

40
60
Time (sec)

80

100

-10

20

40
Time (sec)

60

80

Root locus and Step response for Problem 5.39c

2015 Pearson Education, Inc., Upper Saddle River, NJ. All rights reserved. This publication is protected by Copyright and written permission should be obtained
from the publisher prior to any prohibited reproduction, storage in a retrieval system, or transmission in any form or by any means, electronic, mechanical, photocopying,
recording, or likewise. For information regarding permission(s), write to: Rights and Permissions Department, Pearson Education, Inc., Upper Saddle River, NJ 07458.

5069
40. Golden Nugget Airlines has opened a free bar in the tail of their airplanes
in an attempt to lure customers. In order to automatically adjust for the
sudden weight shift due to passengers rushing to the bar when it rst
opens, the airline is mechanizing a pitch-attitude auto pilot. Figure 5.61
shows the block diagram of the proposed arrangement. We will model the
passenger moment as a step disturbance Mp (s) = M0 =s, with a maximum
expected value for M0 of 0.6.

Figure 5.61: Golden Nugget Airlines Autopilot

(a) What value of K is required to keep the steady-state error in


less than 0.02 rad(= 1 )? (Assume the system is stable.)

to

(b) Draw a root locus with respect to K.


(c) Based on your root locus, what is the value of K when the system
becomes unstable?
(d) Suppose the value of K required for acceptable steady-state behavior
is 600. Show that this value yields an unstable system with roots at
s=

2:9; 13:5; +1:2

6:6j:

(e) You are given a black box with rate gyro written on the side and told
that when installed, it provides a perfect measure of _ , with output
KT _ . Assume K = 600 as in part (d) and draw a block diagram
indicating how you would incorporate the rate gyro into the auto
pilot. (Include transfer functions in boxes.)
(f) For the rate gyro in part (e), sketch a root locus with respect to KT .
(g) What is the maximum damping factor of the complex roots obtainable with the conguration in part (e)?
(h) What is the value of KT for part (g)?

2015 Pearson Education, Inc., Upper Saddle River, NJ. All rights reserved. This publication is protected by Copyright and written permission should be obtained
from the publisher prior to any prohibited reproduction, storage in a retrieval system, or transmission in any form or by any means, electronic, mechanical, photocopying,
recording, or likewise. For information regarding permission(s), write to: Rights and Permissions Department, Pearson Education, Inc., Upper Saddle River, NJ 07458.

5070

CHAPTER 5. THE ROOT-LOCUS DESIGN METHOD


(i) Suppose you are not satised with the steady-state errors and damping ratio of the system with a rate gyro in parts (e) through (h).
Discuss the advantages and disadvantages of adding an integral term
and extra lead networks in the control law. Support your comments
using Matlab or with rough root-locus sketches.
Solution:
(a) Since any error is due to the disturbance Mp , we dene the transfer
function from Mp to :
(s + 3)(s + 10)
(s)
=
Mp (s)
s(s + 10)(s2 + 4s + 5) + K(s + 3)
Using the Final Value Theorem, the steady-state error is
e(1) = lim s
s!0

10Mo
(s) Mo
=
Mp (s) s
K

(10)(0:6)
< 0:02
K

Therefore we need K > 300.


(b) The characteristic equation of the system in Evans form is
1+K

(s + 3)
=0
s(s + 10)(s2 + 4s + 5)

The root locus is plotted below.


Root locus for problem 5.41

Imag Axis

-1

-2

-3

-4
-10

-8

-6

-4

-2

Real Axis

2015 Pearson Education, Inc., Upper Saddle River, NJ. All rights reserved. This publication is protected by Copyright and written permission should be obtained
from the publisher prior to any prohibited reproduction, storage in a retrieval system, or transmission in any form or by any means, electronic, mechanical, photocopying,
recording, or likewise. For information regarding permission(s), write to: Rights and Permissions Department, Pearson Education, Inc., Upper Saddle River, NJ 07458.

5071
(c) To nd the stability boundary, we can do the Routh test or solve the
characteristic equation for the j! crossings. Here, the latter method
is used. The characteristic equation of the system is
(s)

1+K

(s + 3)
s(s + 10)(s2 + 4s + 5)

= s4 + 14s3 + 45s2 + (50 + K)s + 3K = 0


Plugging s = j!, we can write
(j!) = ! 4

45! 2 + 3K + j

14! 3 + (50 + K)! = 0


q
50+K
From the imaginary part, we have ! = 0;
14 . Substituting
these into the real part, we see that the roots are on the imaginary
axis when K = 143:7. So the system goes unstable if K > 143:7.
This agrees with the root locus.
(d) When K = 600, the characteristic equation is
s4 + 14s3 + 45s2 + 650s + 1800 = 0
The roots of the equation are s = 13:5; 2:94; 1:22 6:63:
(e) The output of the rate gyro box would be added at the same spot as
the attitude sensor output.
(f) With the rate feedback, the characteristic equation in Evans form is
1 + KT

s (s +

600s (s + 3)
=0
+ 4s + 5) + 600 (s + 3)

10) (s2

The root locus is shown below.


Root Locus
15
0.84

0.74

0.6

0.42

0.22

0.92

10
0.965

Imag Axis

0.99

30
0

25

20

15

10

0.99
-5

0.965
-10

0.92
0.84
-15
-30

-25

0.74
-20

-15

0.6
-10

0.42

0.22
-5

10

Real Axis

Root locus for Problem 5.40f

2015 Pearson Education, Inc., Upper Saddle River, NJ. All rights reserved. This publication is protected by Copyright and written permission should be obtained
from the publisher prior to any prohibited reproduction, storage in a retrieval system, or transmission in any form or by any means, electronic, mechanical, photocopying,
recording, or likewise. For information regarding permission(s), write to: Rights and Permissions Department, Pearson Education, Inc., Upper Saddle River, NJ 07458.

5072

CHAPTER 5. THE ROOT-LOCUS DESIGN METHOD


(g) From the root locus in Matlab, we can draw a point around and nd
that the maximum damping ratio occurs at s = 3:54 12j with a
max damping of = 0:282.
(h) From the root locus in Matlab, we can immediately get the gain value
at the maximum damping; it is KT = 0:288.
(i) Integral (PI) control would reduce the steady-state error to the moment to zero but would make the damping less and the settling time
longer. A lead network could improve the damping of the response.
The root locus and step responses for the cases with an integrator
and with an integrator+extra lead network are shown below.
Root locus for problem 5.41

Imag Axis

-1

-2

-3

-4
-10

-8

-6

-4

-2

Real Axis

2015 Pearson Education, Inc., Upper Saddle River, NJ. All rights reserved. This publication is protected by Copyright and written permission should be obtained
from the publisher prior to any prohibited reproduction, storage in a retrieval system, or transmission in any form or by any means, electronic, mechanical, photocopying,
recording, or likewise. For information regarding permission(s), write to: Rights and Permissions Department, Pearson Education, Inc., Upper Saddle River, NJ 07458.

5073
41. Consider the instrument servomechanism with the parameters given in
Fig. 5.62. For each of the following cases, draw a root locus with respect
to the parameter K, and indicate the location of the roots corresponding
to your nal design.

Figure 5.62: Control system for Problem 5.41

(a) Lead network : Let


H(s) = 1;

Dc (s) = K

s+z
;
s+p

p
= 6:
z

Select z and K so that the roots nearest the origin (the dominant
roots) yield
0:4;

7;

Kv

2
16 sec
3

(b) Output-velocity (tachometer) feedback : Let


H(s) = 1 + KT s and Dc (s) = K:
Select KT and K so that the dominant roots are in the same location
as those of part (a). Compute Kv . If you can, give a physical reason explaining the reduction in Kv when output derivative feedback
is used.
(c) Lag network : Let
H(s) = 1

and D(s) = K

s+1
:
s+p

Using proportional control, is it possible to obtain a Kv = 12 at


= 0:4? Select K and p so that the dominant roots correspond to the
proportional-control case but with Kv = 100 rather than Kv = 12.

2015 Pearson Education, Inc., Upper Saddle River, NJ. All rights reserved. This publication is protected by Copyright and written permission should be obtained
from the publisher prior to any prohibited reproduction, storage in a retrieval system, or transmission in any form or by any means, electronic, mechanical, photocopying,
recording, or likewise. For information regarding permission(s), write to: Rights and Permissions Department, Pearson Education, Inc., Upper Saddle River, NJ 07458.

5074

CHAPTER 5. THE ROOT-LOCUS DESIGN METHOD


Solution:
(a) Setting p = 6z, the velocity constant is
Kv = lim s
s!0

K(s + z)
1
K
=
2
s + 6z s(s + 51s + 550)
3300

Thus the Kv requirement leads to K


35200: With K = 40000, a
root locus can be drawn with respect to z.
1+z

6s(s2 + 51s + 550) + 40000


=0
s2 (s2 + 51s + 550) + 40000s
Root locus with res pect to z

30

20

Imaginary Ax is

10

-10

-20

-30
-45

-40

-35

-30

-25

-20
Real Axis

-15

-10

-5

Root locus for Problem 5.41(a)


At the point of maximum damping, the values are z = 16:8 and
the dominant roots are at s = 11 13j: So the compensator is
s + 16:8
Dc (s) = 40000
.
s + 100:8
(b) With H(s) = 1 + KT s and Dc (s) = K, the closed-loop transfer
function is
Y
K
= 3
2
R
s + 51s + (550 + KKT )s + K
For this system to have poles at s = 11 13j:, the characteristic
polynomial should be in the form of
(s + p)(s2 + 22s + 290) = s3 + (p + 22)s2 + (22p + 290)s + 290p
Equating the coe cients leads to p = 29, K = 8410, and KT = 0:045.
With these value, the velocity constant is
1
= lim s 1
s!0
Kv

Y
R

1
550 + KKT
=
s2
K

Kv = 9:058

2015 Pearson Education, Inc., Upper Saddle River, NJ. All rights reserved. This publication is protected by Copyright and written permission should be obtained
from the publisher prior to any prohibited reproduction, storage in a retrieval system, or transmission in any form or by any means, electronic, mechanical, photocopying,
recording, or likewise. For information regarding permission(s), write to: Rights and Permissions Department, Pearson Education, Inc., Upper Saddle River, NJ 07458.

5075
The output derivative feedback is acting only when there is a change
in the output. Therefore, for a ramp input, the derivative action will
minimize the deviation from the reference because the input signal
is continuously increasing.
(c) Using proportional control (Dc (s) = K), the velocity constant is
Kv = lim sK
s!0

K
1
=
s(s2 + 51s + 550)
550

Therefore Kv = 12 can be obtained by setting K = 6600. With this


value, the dominant roots are at s = 4:7 11:69j, and = 0:37.
s+1
With Dc (s) = K s+p
, the velocity constant is

Kv = lim s
s!0

K(s + 1)
1
K
=
s + p s(s2 + 51s + 550)
550p

So Kv = 100 can be obtained by setting K


p = 55000. Setting K =
55000p, a root locus can be drawn with the parameter p
1+p

s(s2 + 51s + 550) + 55000(s + 1)


=0
s2 (s2 + 51s + 550)
Root loc us with res pec t to p

20

15

10

Imaginary Axis

-5

-10

-15

-20
-40

-35

-30

-25

-20

-15
Real Ax is

-10

-5

10

Root locus for Problem 5.41(c)


In the plot, the desired pole locations are marked with a dot ( ).
Thus we can choose p = 0:11 to place the poles near the desired
s+1
.
locations. Thus the compensator is Dc (s) = 6050
s + 0:11

2015 Pearson Education, Inc., Upper Saddle River, NJ. All rights reserved. This publication is protected by Copyright and written permission should be obtained
from the publisher prior to any prohibited reproduction, storage in a retrieval system, or transmission in any form or by any means, electronic, mechanical, photocopying,
recording, or likewise. For information regarding permission(s), write to: Rights and Permissions Department, Pearson Education, Inc., Upper Saddle River, NJ 07458.

5076

CHAPTER 5. THE ROOT-LOCUS DESIGN METHOD

Problems and solutions for Section 5.6


42. Plot the loci for the 0 locus or negative K for each of the following:

(a) The examples given in Problem 5.3


(b) The examples given in Problem 5.4
(c) The examples given in Problem 5.5
(d) The examples given in Problem 5.6
(e) The examples given in Problem 5.7
(f) The examples given in Problem 5.8

Solution:

plot a
15

10

Imaginary A xis

-5

-10

-15
-15

-10

-5

0
Real Axis

10

15

(a)
Problem 5.42(a)

2015 Pearson Education, Inc., Upper Saddle River, NJ. All rights reserved. This publication is protected by Copyright and written permission should be obtained
from the publisher prior to any prohibited reproduction, storage in a retrieval system, or transmission in any form or by any means, electronic, mechanical, photocopying,
recording, or likewise. For information regarding permission(s), write to: Rights and Permissions Department, Pearson Education, Inc., Upper Saddle River, NJ 07458.

5077

plot a

plot b

10

20

15
Imaginary Axis

Imaginary Axis

6
4
2
0
-2
-4

10
5
0
-5
-10

-6
-15

-8
-15

-10

-5
Real Axis

10

-15

-10

-5
Real Axis

plot c

10

-2

plot d

1.5
Imaginary Axis

Imaginary Axis

1
0.5
0
-0.5
-1

0.5

-0.5

-1.5
-20

-15

-10
-5
Real Axis

-1

-10

-8

-6

-4
Real Axis

Problem 5.42(b)

plot a

plot b
10
Imaginary Axis

Imaginary Axis

5
0
-5

-5
-4

-3

-2

-1
Real Axis

-6

-4

-2

0
2
Real Axis

plot c

plot d

Imaginary Axis

Imaginary Axis

4
2
1
0
-1

2
0
-2

-2
-3

-4
-1

-0.5
Real Axis

0.5

-1

-0.5
Real Axis

plot e

0.5

plot f
3
Imaginary Axis

3
Imaginary Axis

2
1
0
-1
-2

2
1
0
-1
-2

-3
-0.4

-0.2

0.2
0.4
Real Axis

0.6

0.8

-3
-0.6

-0.4

-0.2

0
Real Axis

0.2

0.4

Problem 5.42(c)

2015 Pearson Education, Inc., Upper Saddle River, NJ. All rights reserved. This publication is protected by Copyright and written permission should be obtained
from the publisher prior to any prohibited reproduction, storage in a retrieval system, or transmission in any form or by any means, electronic, mechanical, photocopying,
recording, or likewise. For information regarding permission(s), write to: Rights and Permissions Department, Pearson Education, Inc., Upper Saddle River, NJ 07458.

5078

CHAPTER 5. THE ROOT-LOCUS DESIGN METHOD

20
10
0
-10
-20

-50
-20

-10

10
Real Axis
plot c

20

30

40

-20

Imaginary Axis

Imaginary Axis

plot b
Imaginary Axis

Imaginary Axis

plot a
50

20
10
0
-10
-20
-20

-10

Imaginary Axis

Imaginary Axis

5
0
-5
-10

-10

-5

-15

-10

10

-5

Real Axis
g
Imaginary Axis

-5
Real Axis
plot f

0.5

-0.5
-10

10

-0.5

Real Axis
plot e
10

0.5

-1

10

-10
Real Axis
plot d

Real Axis

20
10
0
-10
-20
-20

-10

0
Real Axis

10

Problem 5.42(d)

plot b

2
0
-2

Imaginary Ax is

10

2
0
-2

2
0
-2

-4

-4

-4

-6

-6

-6

-8

-8
-10

-5
0
Real Ax is

10

-8
-10

-5
0
Real Ax is

plot d
1

0.8

0.6

0.4

2
0
-2

10

-10

-5
0
Real Ax is

0.2
0
-0.2

-4

-0.4

-6

-0.6

-8
-10
-20

plot e

10

Imaginary Ax is

Imaginary Ax is

plot c

10

Imaginary Ax is

Imaginary Ax is

plot a
10

-0.8
-10

0
Real Ax is

10

20

-3

-2

-1
Real Ax is

Problem 5.42(e)

2015 Pearson Education, Inc., Upper Saddle River, NJ. All rights reserved. This publication is protected by Copyright and written permission should be obtained
from the publisher prior to any prohibited reproduction, storage in a retrieval system, or transmission in any form or by any means, electronic, mechanical, photocopying,
recording, or likewise. For information regarding permission(s), write to: Rights and Permissions Department, Pearson Education, Inc., Upper Saddle River, NJ 07458.

5079

plot b

plot c

0.8

1.5

0.6

0.4
0.2
0
-0.2

Imaginary Axis

Imaginary Axis

Imaginary Axis

plot a
1

2
1
0
-1

-0.4

-2

-0.6

-3

0
-0.5
-1
-1.5

-0.8

-4
-15

-10

-5
Real Axis

-10

plot d

-5
Real Axis

-2
-2

plot e

2
0
-2

Imaginary Axis

Imaginary Axis

0
2
Real Axis

plot f

10

Imaginary Axis

0.5

4
2
0
-2
-4

1
0
-1
-2

-4
-6
-6
-8
-30

-3

-8
-20
-10
Real Axis

-10

-5
0
Real Axis

-4
-5

0
Real Axis

Problem 5.42(f)

2015 Pearson Education, Inc., Upper Saddle River, NJ. All rights reserved. This publication is protected by Copyright and written permission should be obtained
from the publisher prior to any prohibited reproduction, storage in a retrieval system, or transmission in any form or by any means, electronic, mechanical, photocopying,
recording, or likewise. For information regarding permission(s), write to: Rights and Permissions Department, Pearson Education, Inc., Upper Saddle River, NJ 07458.

5080

CHAPTER 5. THE ROOT-LOCUS DESIGN METHOD

43. Suppose you are given the plant


L(s) =

1
;
s2 + (1 + )s + (1 + )

where
is a system parameter that is subject to variations. Use both
positive and negative root-locus methods to determine what variations in
can be tolerated before instability occurs.
Solution:
The characteristic polynomial in Evans form with respect to
1+

is

s+1
=0
s2 + s + 1

The positive and negative root locus are shown below.

Positive root locus

Negative root locus

1.5

0.8
1
0.6

0.4

Imaginary Axis

Imaginary Axis

0.5

0.2

-0.2
-0.5
-0.4

-0.6
-1
-0.8

-1.5
-4

-3

-2

-1
Real Axis

-1
-2

-1.5

-1

-0.5
Real Axis

0.5

Positive(left) and Negative(right) root locus for Problem 5.43


From the root locus, we see that the system is stable for all

>

1.

2015 Pearson Education, Inc., Upper Saddle River, NJ. All rights reserved. This publication is protected by Copyright and written permission should be obtained
from the publisher prior to any prohibited reproduction, storage in a retrieval system, or transmission in any form or by any means, electronic, mechanical, photocopying,
recording, or likewise. For information regarding permission(s), write to: Rights and Permissions Department, Pearson Education, Inc., Upper Saddle River, NJ 07458.

5081
44. Consider the system in Fig. 5.63.

Figure 5.63: Feedback system for Problem 5.44

(a) Use Rouths criterion to determine the regions in the (K1 ; K2 ) plane
for which the system is stable.
(b) Use rltool to verify your answer to part (a).
Solution:

(a) Dene kp = K1 and kI = K1 K2 and the characteristic polynomial is


a(s) = s4 + 1:5s3 + 0:5s2 + kp s + kI
The Routh array for this polynomial is
(s)

= s4 + 1:5s3 + 0:5s2 + kp s + kI
s4 :
s3 :

1
1:5
3 4kp
s2 :
6
9kI
s : kp
3 4kp
s0 :
kI

0:5 kI
kp
kI

For the system to be stable, it is necessary that


kI > 0, kp < 0:75, and 4kp2

3kp + 9kI < 0

The third of these represents a parabola in the [kp; kI ] plane plotted


below. The region of stability is the area under the parabola and
above the kp axis.

2015 Pearson Education, Inc., Upper Saddle River, NJ. All rights reserved. This publication is protected by Copyright and written permission should be obtained
from the publisher prior to any prohibited reproduction, storage in a retrieval system, or transmission in any form or by any means, electronic, mechanical, photocopying,
recording, or likewise. For information regarding permission(s), write to: Rights and Permissions Department, Pearson Education, Inc., Upper Saddle River, NJ 07458.

5082

CHAPTER 5. THE ROOT-LOCUS DESIGN METHOD


Stability region in the kp,kI plane for problem 5.43
0.07

0.06

0.05

kI

0.04

0.03

0.02

0.01

0.1

0.2

0.3

0.4
kp

0.5

0.6

0.7

0.8

Stability region for Problem 5.44(a)


(b) When kI = 0; there is obviously a pole at the origin. For points on
the parabola, consider kp = 3=8 and kI = 1=16: The roots of the
characteristic equation are 1:309; 0:191; and 0:5j:

2015 Pearson Education, Inc., Upper Saddle River, NJ. All rights reserved. This publication is protected by Copyright and written permission should be obtained
from the publisher prior to any prohibited reproduction, storage in a retrieval system, or transmission in any form or by any means, electronic, mechanical, photocopying,
recording, or likewise. For information regarding permission(s), write to: Rights and Permissions Department, Pearson Education, Inc., Upper Saddle River, NJ 07458.

5083

Figure 5.64: Control system for Problem 5.45

45. The block diagram of a positioning servomechanism is shown in Fig. 5.64.


(a) Sketch the root locus with respect to K when no tachometer feedback
is present (KT = 0).
(b) Indicate the root locations corresponding to K = 16 on the locus
of part (a). For these locations, estimate the transient-response parameters tr , Mp , and ts . Compare your estimates to measurements
obtained using the step command in Matlab.
(c) For K = 16, draw the root locus with respect to KT .
(d) For K = 16 and with KT set so that Mp = 0:05( = 0:707), estimate
tr and ts . Compare your estimates to the actual values of tr and ts
obtained using Matlab.
(e) For the values of K and KT in part (d), what is the velocity constant
Kv of this system?
Solution:
(a) When KT = 0, the characteristic equation of the system is
1+K

1
=0
s(s + 2)

The root locus is plotted at the end.


(b) With K = 16, ! n = 4 and = 0:25. Using the design relations, wed
estimate the overshoot to be Mp = 45% and a rise time of 0:45 sec,
and a settling time of 4:6 sec. The values from the step command in
Matlab are Mp = 44:2%; tr = 0:32 sec; and ts = 4:32 sec : The pole
locations are indicated with ( ) on the plot for (a).
(c) The characteristic equation of the system is s2 + (2 + KT )s + K = 0.
With K = 16, it can be written in Evans form as
s
1 + KT 2
=0
s + s + 16
The root locus is shown below.

2015 Pearson Education, Inc., Upper Saddle River, NJ. All rights reserved. This publication is protected by Copyright and written permission should be obtained
from the publisher prior to any prohibited reproduction, storage in a retrieval system, or transmission in any form or by any means, electronic, mechanical, photocopying,
recording, or likewise. For information regarding permission(s), write to: Rights and Permissions Department, Pearson Education, Inc., Upper Saddle River, NJ 07458.

5084

CHAPTER 5. THE ROOT-LOCUS DESIGN METHOD


(d) Use rlocnd on the locus vs KT to nd the KT value that yields
0.7 damping; the locations are marked with ( ). This shows that
KT = 3:66: Using the formulas inside the back cover yields Mp = 5%;
tr = 0:45 sec; and ts = 1:62 sec : The values from the step command
in Matlab are Mp = 4:3%; tr = 0:54 sec; and ts = 1:65 sec :.

Root locus for Problem 5.45(a)

Step response for Problem 5.45(b)

1.5

1
Amplitude

Imaginary Axis

-2

0.5

-4

-6
-6

0
-4

-2

Real Axis

Root locus vs KT , Problem 5.45(c)

3
Time (sec)

step response for Problem 5.45(d)

1.5
0.76

0.86

0.64

0.5

0.34

0.16

1
1 0.985
6
0

-1

0.985

-2

0.94

Amplitude

Imaginary Axis

2 0.94

0.5

-3
0.86
-4
-6

0.76

0.64

0.5

0.34

0.16
0

-5

-4

-3

-2
Real Axis

-1

0.5

1
1.5
Time (sec)

2.5

Plots for Problem 5.45


(e) The velocity constant is
Kv = lim s
s!0

K
K
=
= 2:83
s(s + 2 + KT )
2 + KT

2015 Pearson Education, Inc., Upper Saddle River, NJ. All rights reserved. This publication is protected by Copyright and written permission should be obtained
from the publisher prior to any prohibited reproduction, storage in a retrieval system, or transmission in any form or by any means, electronic, mechanical, photocopying,
recording, or likewise. For information regarding permission(s), write to: Rights and Permissions Department, Pearson Education, Inc., Upper Saddle River, NJ 07458.

5085
46. Consider the mechanical system shown in Fig. 5.65, where g and a0 are
gains. The feedback path containing gs controls the amount of rate feedback. For a xed value of a0 , adjusting g corresponds to varying the
location of a zero in the s-plane.

Figure 5.65: Control system for Problem 5.46

(a) With g = 0 and


complex.

= 1, nd a value for a0 such that the poles are

(b) Fix a0 at this value, and construct a root locus that demonstrates
the eect of varying g.

Solution:

(a) The roots are complex for a0 > 0:25: We select a0 = 1 and the roots
are at s = 0:5 0:866j.
(b) With respect to g, the root-locus form of the characteristic equation
s
is 1 + g 2
= 0: The locus is plotted below.
s +s+1

2015 Pearson Education, Inc., Upper Saddle River, NJ. All rights reserved. This publication is protected by Copyright and written permission should be obtained
from the publisher prior to any prohibited reproduction, storage in a retrieval system, or transmission in any form or by any means, electronic, mechanical, photocopying,
recording, or likewise. For information regarding permission(s), write to: Rights and Permissions Department, Pearson Education, Inc., Upper Saddle River, NJ 07458.

5086

CHAPTER 5. THE ROOT-LOCUS DESIGN METHOD


Rootlocus for problem 5.46

0.8

0.84

0.92

0.74

0.6

0.42

0.22

0.6
0.965
0.4
0.99

Imag Axis

0.2

2
0

1.75

1.5

1.25

0.75

0.5

0.25

-0.2
0.99
-0.4
0.965
-0.6

-0.8
-2

0.92

0.84
-1.5

0.74

0.6

-1

0.42
-0.5

0.22
0

Real Axis

47. Sketch the root locus with respect to K for the system in Fig. 5.66 using
the Pad(1,1) approximation and the rst-order lag approximation. For
both approximations, what is the range of values of K for which the system
is unstable?(Note: The material to answer this question is contained in
Appendix W5.6.3 discussed in www.FPE7e.com.)

Figure 5.66: Control system for Problem 5.47

Solution:
Matlab cannot directly plot a root locus for a transcendental function.
From the Appendix W5.6.3, we see that the Pad(1,1) approximation for

2015 Pearson Education, Inc., Upper Saddle River, NJ. All rights reserved. This publication is protected by Copyright and written permission should be obtained
from the publisher prior to any prohibited reproduction, storage in a retrieval system, or transmission in any form or by any means, electronic, mechanical, photocopying,
recording, or likewise. For information regarding permission(s), write to: Rights and Permissions Department, Pearson Education, Inc., Upper Saddle River, NJ 07458.

5087
e

is
e

1 (s=2)
=
1 + (s=2)

s 2
;
s+2

and the rst-order lag approximation (Pad(0,1)) is


e

1
:
s+1

With the Pad(1,1) approximation, a locus valid for small values of s can
be plotted, as shown below by the red curve. The rlocnd routine is used
by placing the cursor on the j! axis to nd the maximum value of K at
the instability boundary. This yields Kmax = 2 for the Pad(1,1). The
locus for the rst-order lag is also shown in green; however, it produces
a locus with two branches going to innity at s = 1=2: thus there is
no value of K that produces instability for this approximation for a rst
order system. This demonstrates a limitation for this approximation of
the delay. However, for higher order systems, the rst order lag can be
useful.

Root loci f or problem 5.46 with the (1,1)and (0,1) Pade aproxim ates
3

Imaginary Axis

-1

-2

-3
-6

-5

-4

-3

-2
Real Axis

-1

Solutions for Problem 5.47

2015 Pearson Education, Inc., Upper Saddle River, NJ. All rights reserved. This publication is protected by Copyright and written permission should be obtained
from the publisher prior to any prohibited reproduction, storage in a retrieval system, or transmission in any form or by any means, electronic, mechanical, photocopying,
recording, or likewise. For information regarding permission(s), write to: Rights and Permissions Department, Pearson Education, Inc., Upper Saddle River, NJ 07458.

5088

CHAPTER 5. THE ROOT-LOCUS DESIGN METHOD

48. Prove that the plant G(s) = 1=s3 cannot be made unconditionally stable
if pole cancellation is forbidden.
Solution:
The angles of departure from a triple pole are 180 and 60 for the
negative locus and 0 and 120 for the positive locus. In either case, at
least one pole starts out into the right-half plane. Such a system must be
conditionally stable for it will be unstable if the gain is small enough.

2015 Pearson Education, Inc., Upper Saddle River, NJ. All rights reserved. This publication is protected by Copyright and written permission should be obtained
from the publisher prior to any prohibited reproduction, storage in a retrieval system, or transmission in any form or by any means, electronic, mechanical, photocopying,
recording, or likewise. For information regarding permission(s), write to: Rights and Permissions Department, Pearson Education, Inc., Upper Saddle River, NJ 07458.

5089
49. For the equation 1 + KG(s) where,
G(s) =

1
;
s(s + p)[(s + 1)2 + 4]

use Matlab to examine the root locus as a function of K for p in the range
from p = 1 to p = 10, making sure to include the point p = 2.
Solution:
The root loci for four values are given in the gure. The point is that the
locus for p = 2 has multiple roots at a complex value of s:

Problem 5.49

Problem 5.49
6
4

Imag Axis

Imag Axis

p=2

p=1

0
-2
-4

0
-2
-4

-6
-6
-5

-6

-4

Real Axis

Problem 5.49

Problem 5.49

10

p=5

p = 10

20

10
Imag Axis

Imag Axis

-2

Real Axis

-10

-5

-20
-10
-10

-5

Real Axis

-20

-10

10

20

Real Axis

Solutions for Problem 5.49

2015 Pearson Education, Inc., Upper Saddle River, NJ. All rights reserved. This publication is protected by Copyright and written permission should be obtained
from the publisher prior to any prohibited reproduction, storage in a retrieval system, or transmission in any form or by any means, electronic, mechanical, photocopying,
recording, or likewise. For information regarding permission(s), write to: Rights and Permissions Department, Pearson Education, Inc., Upper Saddle River, NJ 07458.

You might also like